You are on page 1of 58

Module 9 Infection

1) The nurse is concerned that a client with bowel and bladder dysfunction is at risk for developing an
infection. Which actions should the nurse take to help reduce this client's risk for developing an
infection?
Select all that apply.
A) Turn and reposition the client every 2 hours.
B) Monitor intake and output.
C) Provide hygienic care after episodes of incontinence.
D) Use standard precautions when handling linen after episodes of incontinence.
E) Cover wounds with antibiotic ointment and sterile gauze.
Answer: B, C, D
Explanation: A) Turning and repositioning is an intervention to maintain tissue integrity. Monitoring
for dehydration will help reduce the risk of developing an infection because of bowel and bladder
dysfunction. Providing hygienic care after episodes of bowel or bladder incontinence will ensure that
the skin remains intact, reducing the risk of infection. Using proper biohazard precautions after
episodes of incontinence will reduce the risk of transmitting an infection. Covering wounds with
antibiotic ointment and sterile gauze is an intervention to maintain tissue integrity. Page Ref: 526
Cognitive Level: Applying Client Need: Safe and Effective Care Environment Nursing Process:
Implementation Learning Outcome: 2. Examine the relationship between infection and other
concepts/systems.

2) A nurse is planning an in-service on preventing infection for the hospital staff nurses on a medical-
surgical unit. Which nursing intervention is most effective in reducing the risk of infections?
A) Raising the temperature in the client's room
B) Assessing vital signs once daily
C) Wearing a mask for client care
D) Performing hand hygiene
Answer: D
Explanation: A) Hand hygiene is always the first and best way to stop the spread of microorganisms,
which cause infections. Raising the temperature in a client's room would contribute to the growth of
microorganisms. Assessing vital signs is important but should be done more frequently than once
daily. Wearing a mask for all clients is not practical and is unnecessary unless a microorganism is
airborne and the client is in isolation. Page Ref: 531 Cognitive Level: Applying Client Need: Safe and
Effective Care Environment Nursing Process: Planning Learning Outcome: 1. Summarize the
physiology of the immune system related to infection prevention.

3) The nurse is assessing an 80-year-old client who is recovering following a cholecystectomy. Which
factor would increase this client's susceptibility to infection?
Select all that apply.
A) Intact mucous membranes
B) Surgical incision
C) Dry skin
D) Active bowel sounds
Answer: B, C, D
Explanation: A) Certain factors will increase a client's susceptibility to microorganism growth and
infection. This client is 80 years old and has a surgical incision, so the first line of defense, the skin, is
not intact. Active bowel sounds, dry skin, and intact mucous membranes are factors that help defend
the body against infection. Page Ref: 526 Cognitive Level: Analyzing Client Need: Safe and Effective
Care Environment Nursing Process: Assessment Learning Outcome: 3. Identify commonly occurring
alterations in the immune system that increase the risk for or occurrence of infection and their related
4) The nurse is caring for a client who is being discharged following an appendectomy. Which
instruction is the most important for the nurse to teach this client regarding wound healing?
A) "Thoroughly irrigate the wound with hydrogen peroxide once a day."
B) "Apply a lubricating lotion to the edges of the wound twice a day."
C) "Add more fruits and vegetables to your diet."
D) "Notify the physician if you notice swelling, warmth, or tenderness at the wound site."
Answer: D
Explanation: A) A client being discharged with an open surgical wound has to be instructed on the
detection of infection, as the skin is the first line of defense. Signs such as edema, heat, and
tenderness would indicate a local infection. Increasing fruits and vegetables would increase vitamin C,
which helps with wound healing, but more protein would be the best choice. Irrigating with hydrogen
peroxide would break down good granulating tissue, so this also would not increase healing. Applying
lubricating lotion to the edges of a wound would impede the healing process. Page Ref: 551
Cognitive Level: Applying Client Need: Safe and Effective Care Environment Nursing Process:
Implementation Learning Outcome: 4. Differentiate common assessment procedures used to
examine for the presence of infection client across the life span.

5) The nurse is caring for a client who is experiencing a systemic infection after a total knee
replacement. Which diagnostic tests will be used to validate the presence of this infection?
Select all that apply.
A) Serum electrolyte levels
B) Urinalysis
C) White blood cell differential
D) White blood cell count
E) Wound culture
Answer: B, C, D, E
Explanation: A) Serum electrolyte levels are not used to determine the presence of a systemic
infection. Urinalysis is used to assess for the presence of bacteria or blood in the urine. An elevated
WBC and 15% bands are indicative of an infection. Wound cultures are used to identify probable
microorganisms. Page Ref: 555-557 Cognitive Level: Analyzing Client Need: Physiological Integrity
Nursing Process: Planning Learning Outcome: 5. Describe diagnostic and laboratory tests to
determine the individual's infection status.

6) The nurse is teaching a class on infection control. Which nursing measure is most appropriate in
breaking a link in the chain of infection?
Select all that apply.
A) Place contaminated linens in a paper bag.
B) Use personal protective equipment (PPE).
C) Cover one's cough by placing the mouth in the elbow.
D) Wear sterile gloves for client care.
Answer: B, C
Explanation: A) Placing linens in a paper bag would allow germs to come out through the bag, and
the linen would act as a fomite, thus allowing the chain to continue. PPE, according to OSHA
standards, has to be used whenever the situation dictates. Covering one's mouth when coughing
prevents airborne droplets from escaping into the air for others to contract in the chain of infection.
Gloves have to be worn but are to be changed between client, and hands are to be washed.
Page Ref: 544-549 Cognitive Level: Applying Client Need: Safe and Effective Care Environment
Nursing Process: Implementation Learning Outcome: 6. Explain management of immune health and
prevention of infection.
7) The nurse is teaching a child care class for mothers of young children. What should the nurse teach
as being the most common mode of transmission of infectious disease?
A) Children who are playing board games
B) Children who are sitting together eating meals
C) Children who are playing with the same toy
D) Children who don't wash their hands after using the bathroom
Answer: D
Explanation: A) The fecal-oral and respiratory routes are the most common sources of transmission
in children. Microorganisms might be left on toys that children share, but this is not the most
common mode of transmission of infectious diseases. Playing with board games will not transmit
infectious disease. Eating together will not transmit infectious disease. Poor hand hygiene is a
common source of transmission. Page Ref: 527 Cognitive Level: Applying Client Need: Health
Promotion and Maintenance Nursing Process: Implementation Learning Outcome: 7. Demonstrate
the nursing process in providing culturally competent and caring interventions across the life span for
individuals with infection.

8) A pediatric client is receiving antibiotics for the treatment of Staphylococcus aureus. Which
nursing interventions are priorities when caring for this client?
Select all that apply.
A) Encourage adequate fluid intake.
B) Monitor for allergic reaction.
C) Assess renal and liver function.
D) Obtain a baseline electrocardiogram.
E) Monitor vital signs.
Answer: A, B, C, E
Explanation: A) Nursing interventions to support antibiotic therapy include encouraging an adequate
fluid intake, monitoring for manifestations of an allergic reaction, assessing renal and hepatic
function, and assessing vital signs. A baseline electrocardiogram is appropriate for antimalarial
medication. Page Ref: 541 Cognitive Level: Applying Client Need: Physiological Integrity Nursing
Process: Implementation Learning Outcome: 8. Compare and contrast common independent and
collaborative interventions for clients with infection.
Exemplar 9.1 Cellulitis

1) A nurse is assessing a client who has recently returned from a camping trip. The client is being seen
for edema in the right foot. When assessing the foot, the nurse notes a sore on the foot and suspects
cellulitis. Which further data will the nurse assess to support the suspicion?
A) BUN and creatinine
B) Breath sounds
C) Blood cultures
D) Redness, pain, and drainage at the site
Answer: D
Explanation: A) Classic signs of cellulitis are swelling, pain, redness, and possibly drainage at the site.
BUN, creatinine, and blood cultures are ordered by the physician and are not nursing assessments.
Breath sounds are not indicated at present, as the infection is in the foot. Blood cultures are ordered
by the physician and are not nursing assessments.
B) Classic signs of cellulitis are swelling, pain, redness, and possibly drainage at the site. BUN,
creatinine, and blood cultures are ordered by the physician and are not nursing assessments. Breath
sounds are not indicated at present, as the infection is in the foot. Blood cultures are ordered by the
physician and are not nursing assessments.
C) Classic signs of cellulitis are swelling, pain, redness, and possibly drainage at the site. BUN,
creatinine, and blood cultures are ordered by the physician and are not nursing assessments. Breath
sounds are not indicated at present, as the infection is in the foot. Blood cultures are ordered by the
physician and are not nursing assessments.
D) Classic signs of cellulitis are swelling, pain, redness, and possibly drainage at the site. BUN,
creatinine, and blood cultures are ordered by the physician and are not nursing assessments. Breath
sounds are not indicated at present, as the infection is in the foot. Blood cultures are ordered by the
physician and are not nursing assessments.
Page Ref: 561
Cognitive Level: Applying
Client Need: Safe and Effective Care Environment
Nursing Process: Assessment
Learning Outcome: 1. Describe the pathophysiology, etiology, clinical manifestations, and direct and
indirect causes of cellulitis.
2) The nurse is teaching a group of adolescents at a local high school about skin infections. Which
students are at risk for developing cellulitis?
Select all that apply.
A) The female student who plucks her eyebrows
B) The student with diabetes
C) The student practicing hand hygiene
D) The student who squeezes pimples
E) The student who engages in contact sports
Answer: A, B, D
Explanation: A) The student who plucks hair risks an infected hair follicle, which can lead to
cellulitis, as can squeezing pimples. The student with diabetes is at risk for foot lesions, which can
lead to cellulitis if left untreated. Hand hygiene does not pose a risk for cellulitis. Contact sports do
not pose a risk for cellulitis.
B) The student who plucks hair risks an infected hair follicle, which can lead to cellulitis, as can
squeezing pimples. The student with diabetes is at risk for foot lesions, which can lead to cellulitis if
left untreated. Hand hygiene does not pose a risk for cellulitis. Contact sports do not pose a risk for
cellulitis.
C) The student who plucks hair risks an infected hair follicle, which can lead to cellulitis, as can
squeezing pimples. The student with diabetes is at risk for foot lesions, which can lead to cellulitis if
left untreated. Hand hygiene does not pose a risk for cellulitis. Contact sports do not pose a risk for
cellulitis.
D) The student who plucks hair risks an infected hair follicle, which can lead to cellulitis, as can
squeezing pimples. The student with diabetes is at risk for foot lesions, which can lead to cellulitis if
left untreated. Hand hygiene does not pose a risk for cellulitis. Contact sports do not pose a risk for
cellulitis.
E) The student who plucks hair risks an infected hair follicle, which can lead to cellulitis, as can
squeezing pimples. The student with diabetes is at risk for foot lesions, which can lead to cellulitis if
left untreated. Hand hygiene does not pose a risk for cellulitis. Contact sports do not pose a risk for
cellulitis.
Page Ref: 560-564
Cognitive Level: Analyzing
Client Need: Health Promotion and Maintenance
Nursing Process: Evaluation
Learning Outcome: 2. Identify risk factors and prevention methods associated with cellulitis.
3) An adolescent is diagnosed with cellulitis from picking the scabs on healing facial pimples. The
mother scolds the child for eating too many oily foods that cause the acne. What should the nurse
teach the family?
A) Antibiotics are the best way to prevent cellulitis.
B) Oily foods do not cause pimples or cellulitis.
C) Consumption of oily foods poses an increased risk for cellulitis.
D) Popping the pimples spreads the germs over the face.
Answer: B
Explanation: A) Foods do not cause acne and, therefore, cellulitis. Antibiotics are the best way to
treat, not prevent, cellulitis. Popping pimples drives the infection deeper into the skin and may cause
more serious infections. Page Ref: 560-564 Cognitive Level: Applying Client Need: Health
Promotion and Maintenance Nursing Process: Implementation Learning Outcome: 3. Illustrate the
nursing process in providing culturally competent care across the life span for individuals with
cellulitis.
4) The nursing is caring for a client who is hospitalized for cellulitis of the foot. Which nursing
diagnoses should the nurse use to plan this client's care?
Select all that apply.
A) Social Isolation related to skin infection
B) Altered Skin Integrity related to skin infection
C) Acute Pain related to skin infection
D) Disturbed Sleep Pattern related to skin infection
E) Powerlessness related to inability to control the infection
Answer: B, C
Explanation: A) Clients with cellulitis have Altered Skin Integrity and will also have Acute Pain at the
site. Controlling pain is a priority. Disturbed Sleep Pattern, Social Isolation, and Powerlessness are
not supported by the information given.
B) Clients with cellulitis have Altered Skin Integrity and will also have Acute Pain at the site.
Controlling pain is a priority. Disturbed Sleep Pattern, Social Isolation, and Powerlessness are not
supported by the information given.
C) Clients with cellulitis have Altered Skin Integrity and will also have Acute Pain at the site.
Controlling pain is a priority. Disturbed Sleep Pattern, Social Isolation, and Powerlessness are not
supported by the information given.
D) Clients with cellulitis have Altered Skin Integrity and will also have Acute Pain at the site.
Controlling pain is a priority. Disturbed Sleep Pattern, Social Isolation, and Powerlessness are not
supported by the information given.
E) Clients with cellulitis have Altered Skin Integrity and will also have Acute Pain at the site.
Controlling pain is a priority. Disturbed Sleep Pattern, Social Isolation, and Powerlessness are not
supported by the information given.
Page Ref: 563
Cognitive Level: Analyzing
Client Need: Safe and Effective Care Environment
Nursing Process: Planning
Learning Outcome: 4. Formulate priority nursing diagnoses appropriate for an individual with
cellulitis.
5) The nurse is preparing discharge instructions for an older client with cellulitis of the leg. What
should the nurse teach the client and family regarding ways to prevent this health problem from
occurring in the future?
A) "Keep the client off her leg as much as possible."
B) "Bring the client to the doctor if there is a fall."
C) "Maintain a healthy diet."
D) "Do not allow the client to walk without assistance."
Answer: C
Explanation: A) The elderly are susceptible to cellulitis, as their immune system and skin are
compromised as they age. The best preventive measure to maintain skin integrity and a healthier
immune system is good nutrition. Exercise is a good way to promote health, including walking. There
is no data to support that the client needs assistance to ambulate. Cellulitis can occur on any part of
the body, so preventing the client from using body parts, such as the legs, is not reasonable and does
not prevent cellulitis. The nurse would instruct the family regarding signs and symptoms of
developing cellulitis after a fall. It is not always necessary to bring the client to the doctor for a fall if
no injuries were sustained.
B) The elderly are susceptible to cellulitis, as their immune system and skin are compromised as they
age. The best preventive measure to maintain skin integrity and a healthier immune system is good
nutrition. Exercise is a good way to promote health, including walking. There is no data to support
that the client needs assistance to ambulate. Cellulitis can occur on any part of the body, so preventing
the client from using body parts, such as the legs, is not reasonable and does not prevent cellulitis.
The nurse would instruct the family regarding signs and symptoms of developing cellulitis after a fall.
It is not always necessary to bring the client to the doctor for a fall if no injuries were sustained.
C) The elderly are susceptible to cellulitis, as their immune system and skin are compromised as they
age. The best preventive measure to maintain skin integrity and a healthier immune system is good
nutrition. Exercise is a good way to promote health, including walking. There is no data to support
that the client needs assistance to ambulate. Cellulitis can occur on any part of the body, so preventing
the client from using body parts, such as the legs, is not reasonable and does not prevent cellulitis.
The nurse would instruct the family regarding signs and symptoms of developing cellulitis after a fall.
It is not always necessary to bring the client to the doctor for a fall if no injuries were sustained.
D) The elderly are susceptible to cellulitis, as their immune system and skin are compromised as they
age. The best preventive measure to maintain skin integrity and a healthier immune system is good
nutrition. Exercise is a good way to promote health, including walking. There is no data to support
that the client needs assistance to ambulate. Cellulitis can occur on any part of the body, so preventing
the client from using body parts, such as the legs, is not reasonable and does not prevent cellulitis.
The nurse would instruct the family regarding signs and symptoms of developing cellulitis after a fall.
It is not always necessary to bring the client to the doctor for a fall if no injuries were sustained.
Page Ref: 561
Cognitive Level: Applying
Client Need: Health Promotion and Maintenance
Nursing Process: Implementation
Learning Outcome: 6. Plan evidence-based care for an individual with cellulitis and the family in
collaboration with other members of the healthcare team.

6) The nurse is teaching a client with cellulitis about ways to avoid future infections. Which client
statements indicate that teaching has been effective?
Select all that apply.
A) "I should use antibiotic soap to cleanse the wounds."
B) "I should eat a lot of rice to increase my intake of vitamin C."
C) "I will contact the doctor if I have a temperature of 99.5°F or higher."
D) "I will not swim in lakes."
E) "I can stop taking the antibiotics when the swelling subsides."
Answer: A, D
Explanation: A) It is recommended that the client wash affected areas with an antibacterial soap. The
water in lakes can be contaminated and can cause an infection. Vitamin C-rich foods include citrus
fruits and strawberries, but not rice. The healthcare provider should be contacted for a body
temperature of 101°F or higher. Antibiotics are taken for the full course ordered.
B) It is recommended that the client wash affected areas with an antibacterial soap. The water in lakes
can be contaminated and can cause an infection. Vitamin C-rich foods include citrus fruits and
strawberries, but not rice. The healthcare provider should be contacted for a body temperature of
101°F or higher. Antibiotics are taken for the full course ordered.
C) It is recommended that the client wash affected areas with an antibacterial soap. The water in lakes
can be contaminated and can cause an infection. Vitamin C-rich foods include citrus fruits and
strawberries, but not rice. The healthcare provider should be contacted for a body temperature of
101°F or higher. Antibiotics are taken for the full course ordered.
D) It is recommended that the client wash affected areas with an antibacterial soap. The water in lakes
can be contaminated and can cause an infection. Vitamin C-rich foods include citrus fruits and
strawberries, but not rice. The healthcare provider should be contacted for a body temperature of
101°F or higher. Antibiotics are taken for the full course ordered.
E) It is recommended that the client wash affected areas with an antibacterial soap. The water in lakes
can be contaminated and can cause an infection. Vitamin C-rich foods include citrus fruits and
strawberries, but not rice. The healthcare provider should be contacted for a body temperature of
101°F or higher. Antibiotics are taken for the full course ordered.
Page Ref: 561
Cognitive Level: Applying
Client Need: Health Promotion and Maintenance
Nursing Process: Evaluation
Learning Outcome: 7. Evaluate expected outcomes for an individual with cellulitis.
7) A client receiving intravenous antibiotics for 3 days as treatment for cellulitis is being prepared for
discharge. Which discharge order should the nurse anticipate for this client?
A) Low-sodium diet prescribed
B) Home healthcare aide for the client
C) Oral antibiotics to be continued at home
D) Orders for evaluation by physical therapy
Answer: C
Explanation: A) Antibiotics should be taken for 10 days. A low-sodium diet is not indicated for
cellulitis. The client may or may not need a home health aide. There is no evidence to suggest the
client needs home physical therapy.
B) Antibiotics should be taken for 10 days. A low-sodium diet is not indicated for cellulitis. The client
may or may not need a home health aide. There is no evidence to suggest the client needs home
physical therapy.
C) Antibiotics should be taken for 10 days. A low-sodium diet is not indicated for cellulitis. The client
may or may not need a home health aide. There is no evidence to suggest the client needs home
physical therapy.
D) Antibiotics should be taken for 10 days. A low-sodium diet is not indicated for cellulitis. The client
may or may not need a home health aide. There is no evidence to suggest the client needs home
physical therapy.
Page Ref: 563
Cognitive Level: Applying
Client Need: Physiological Integrity
Nursing Process: Implementation
Learning Outcome: 5. Summarize therapies used by interdisciplinary teams in the collaborative care
of an individual with cellulitis.
8) The nurse is providing discharge instructions to a client recovering from cellulitis. Which client
statement indicates that this teaching has been effective?
A) "I will monitor for signs of infection such as fever, chills, malaise, and redness or tenderness at the
site."
B) "If the lesion looks healed, I will stop taking the antibiotics so that I will not develop resistance to
antibiotics."
C) "If pustules develop, I will squeeze the lesion to remove the pus."
D) "Drainage from the site is an expected finding, and I should not be concerned."
Answer: A
Explanation: A) The client will need to be taught to monitor for the signs and symptoms of infection.
Infection may be manifested by fever, chills, erythema, tenderness, and drainage at the site, especially
if it is cloudy or serous. Changes in the color, amount, and odor from drainage are cause for concern
and should be monitored. The physician must be notified if these symptoms occur. Pustules are never
to be squeezed open, as there is risk of greater infection. Stopping antibiotics before the doses are
complete causes resistance.
B) The client will need to be taught to monitor for the signs and symptoms of infection. Infection may
be manifested by fever, chills, erythema, tenderness, and drainage at the site, especially if it is cloudy
or serous. Changes in the color, amount, and odor from drainage are cause for concern and should be
monitored. The physician must be notified if these symptoms occur. Pustules are never to be squeezed
open, as there is risk of greater infection. Stopping antibiotics before the doses are complete causes
resistance.
C) The client will need to be taught to monitor for the signs and symptoms of infection. Infection may
be manifested by fever, chills, erythema, tenderness, and drainage at the site, especially if it is cloudy
or serous. Changes in the color, amount, and odor from drainage are cause for concern and should be
monitored. The physician must be notified if these symptoms occur. Pustules are never to be squeezed
open, as there is risk of greater infection. Stopping antibiotics before the doses are complete causes
resistance.
D) The client will need to be taught to monitor for the signs and symptoms of infection. Infection may
be manifested by fever, chills, erythema, tenderness, and drainage at the site, especially if it is cloudy
or serous. Changes in the color, amount, and odor from drainage are cause for concern and should be
monitored. The physician must be notified if these symptoms occur. Pustules are never to be squeezed
open, as there is risk of greater infection. Stopping antibiotics before the doses are complete causes
resistance.
Page Ref: 563
Cognitive Level: Analyzing
Client Need: Health Promotion and Maintenance
Nursing Process: Evaluation
Learning Outcome: 7. Evaluate expected outcomes for an individual with cellulitis.
9) A home health nurse manager is instructing a new staff regarding evidence-based practices of
wound management. Which will the manager's teaching include?
Select all that apply.
A) "Wounds should be kept dry and should not be covered until a scab forms."
B) "Allowing a wound to remain dry or applying a dry wound covering may slow the healing process."
C) "Wound covering helps maintain a dry environment, decreasing the chance of infection such as
cellulitis."
D) "Proper wound moisture management can reduce pain and improve the cosmetic outcome."
E) "Three methods for proper wound management include cleansing techniques, proper wound
covering, and ointment application."
Answer: B, D, E
Explanation: A) Individuals with open wounds are more susceptible to contracting a skin infection
such as cellulitis. Many believe wounds should be kept dry and should not be covered until a scab
forms. However, this slows wound healing and leaves the wound exposed to potential pathogens if the
scab comes off. Wound covering helps to maintain a moist environment, decreasing the chance of
infection such as cellulitis. The three methods of proper moist wound management include cleansing
techniques, proper wound covering, and ointment application. Proper wound moisture management
also reduces pains and improves the cosmetic outcome.
B) Individuals with open wounds are more susceptible to contracting a skin infection such as
cellulitis. Many believe wounds should be kept dry and should not be covered until a scab forms.
However, this slows wound healing and leaves the wound exposed to potential pathogens if the scab
comes off. Wound covering helps to maintain a moist environment, decreasing the chance of infection
such as cellulitis. The three methods of proper moist wound management include cleansing
techniques, proper wound covering, and ointment application. Proper wound moisture management
also reduces pains and improves the cosmetic outcome.
C) Individuals with open wounds are more susceptible to contracting a skin infection such as
cellulitis. Many believe wounds should be kept dry and should not be covered until a scab forms.
However, this slows wound healing and leaves the wound exposed to potential pathogens if the scab
comes off. Wound covering helps to maintain a moist environment, decreasing the chance of infection
such as cellulitis. The three methods of proper moist wound management include cleansing
techniques, proper wound covering, and ointment application. Proper wound moisture management
also reduces pains and improves the cosmetic outcome.
D) Individuals with open wounds are more susceptible to contracting a skin infection such as
cellulitis. Many believe wounds should be kept dry and should not be covered until a scab forms.
However, this slows wound healing and leaves the wound exposed to potential pathogens if the scab
comes off. Wound covering helps to maintain a moist environment, decreasing the chance of infection
such as cellulitis. The three methods of proper moist wound management include cleansing
techniques, proper wound covering, and ointment application. Proper wound moisture management
also reduces pains and improves the cosmetic outcome.
E) Individuals with open wounds are more susceptible to contracting a skin infection such as
cellulitis. Many believe wounds should be kept dry and should not be covered until a scab forms.
However, this slows wound healing and leaves the wound exposed to potential pathogens if the scab
comes off. Wound covering helps to maintain a moist environment, decreasing the chance of infection
such as cellulitis. The three methods of proper moist wound management include cleansing
techniques, proper wound covering, and ointment application. Proper wound moisture management
also reduces pains and improves the cosmetic outcome.
Page Ref: 529
Cognitive Level: Applying
Client Need: Physiological Integrity
Nursing Process: Implementation
Learning Outcome: 3. Illustrate the nursing process in providing culturally competent care across the
life span for individuals with cellulitis.
10) A nurse is caring for a client diagnosed with Erysipelas whose lower extremities are bright red and
raised with well-defined borders. (See image.) The causative organism of this condition is likely:

A) Group B streptococcus.
B) Group A streptococcus.
C) Staphylococcus aureus.
D) Candida albicans.
Answer: B
Explanation: A) Erysipelas, a superficial cellulitis of the skin caused by group A streptococcus, usually
affects the lower extremities or the face. The involved area is bright red and raised with well-defined
borders.
B) Erysipelas, a superficial cellulitis of the skin caused by group A streptococcus, usually affects the
lower extremities or the face. The involved area is bright red and raised with well-defined borders.
C) Erysipelas, a superficial cellulitis of the skin caused by group A streptococcus, usually affects the
lower extremities or the face. The involved area is bright red and raised with well-defined borders.
D) Erysipelas, a superficial cellulitis of the skin caused by group A streptococcus, usually affects the
lower extremities or the face. The involved area is bright red and raised with well-defined borders.
Page Ref: 560
Cognitive Level: Understanding
Client Need: Physiological Integrity
Nursing Process: Assessment
Learning Outcome: 1. Describe the pathophysiology, etiology, clinical manifestations, and direct and
indirect causes of cellulitis.
Exemplar 9.2 Conjunctivitis

1) The nurse is assessing a 20-year-old college student who is experiencing red, swollen eyes that are
oozing a yellowish drainage. The client is complaining of photophobia. What is the most appropriate
question for the nurse to ask the client?
A) "Have you had extra caffeine this week?"
B) "Did you get sand in your eye recently?"
C) "Have you been exposed to HIV?"
D) "Have any of your friends experienced these symptoms?"
Answer: D
Explanation: A) The client is exhibiting signs and symptoms of conjunctivitis. The nurse should
explore ways in which the client may have been exposed. Most cases of conjunctivitis are spread by
hand to eye contact. Exposure to HIV, sand in the eye, and caffeine are not known causes of
conjunctivitis and would not be appropriate questions to ask this client to determine the cause of the
symptoms. Page Ref: 566 Cognitive Level: Applying Client Need: Physiological Integrity Nursing
Process: Assessment Learning Outcome: 1. Describe the pathophysiology, etiology, clinical
manifestations, and direct and indirect causes of conjunctivitis.

2) A client is being treated for trachoma after returning from a safari trip to Africa. Which potential
complication should the nurse consider when planning this client's care?
A) Scarring of the cornea
B) Eye muscle weakness
C) Damaged iris
D) Retinal detachment
Answer: A
Explanation: A) Trachoma is a chronic form of conjunctivitis that causes the formation of granulation
tissue that is abraded by the lashes, leading to scarring of the cornea and eventual blindness. Page
Ref: 567 Cognitive Level: Applying Client Need: Physiological Integrity Nursing Process: Planning
Learning Outcome: 2. Identify risk factors and prevention methods associated with conjunctivitis.

3) The nurse is assessing clients in the eye clinic who have come to be seen for eye infections. For
which client with conjunctivitis should the nurse be most concerned?
A) The client from Iran
B) The client from Brazil
C) The client from New York City
D) The client from Florida
Answer: A
Explanation: A) Trachoma, a chronic form of conjunctivitis, is endemic in poor, undeveloped
countries, especially where there are water shortages, numerous flies, and crowded living conditions.
It can eventually cause blindness, so the nurse would be most concerned about the client from Iran.
None of the other clients lives in an area where the disease is endemic. Page Ref: 567 Cognitive Level:
Analyzing Client Need: Safe and Effective Care Environment Nursing Process: Assessment Learning
Outcome: 3. Illustrate the nursing process in providing culturally competent care across the life span
for individuals with conjunctivitis.

4) The nurse is planning care for a client with conjunctivitis. Which client statement supports the
nursing diagnosis of Risk for Altered Vision?
A) "I have had this infection for 3 days."
B) "My mother is blind from retinopathy."
C) "This is the fourth eye infection I have had in the last 6 months."
D) "I think I caught the infection from my child."
Answer: C
Explanation: A) The concern for this client is the repeated infections that could be a chronic form of
conjunctivitis that causes eventual blindness. Having the infection for 3 days is concerning but not as
much as the potentially chronic form of the disease. Many adults catch "pink eye" from a child, but
that is not usually the form that causes blindness. The client's mother has blindness from a source
other than conjunctivitis. Page Ref: 568 Cognitive Level: Analyzing Client Need: Safe and Effective
Care Environment Nursing Process: Evaluation Learning Outcome: 4. Formulate priority nursing
diagnoses appropriate for an individual with conjunctivitis.

5) The nurse is teaching a mother how to administer optical antibiotics to her child who has
conjunctivitis. Which statement made by the mother indicates teaching has been effective?
A) "I will drop the medication onto the eyeball."
B) "I will wait 10 seconds between drops."
C) "I will wash my hands before instilling the medication."
D) "I will rub the eye with a cotton ball after I administer the medication."
Answer: C
Explanation: A) Teach the client to wash hands thoroughly before and after instilling eye
medications. Handwashing is the single most important means of preventing transmission of
infection. Medication is dropped into the lower conjunctival sac and should not be rubbed after
instillation. The time between drops is 1-5 minutes, depending on the type of medication. The
medication is not dropped on the eyeball but in the conjunctival sac of the lower lid. Page Ref: 569
Cognitive Level: Analyzing Client Need: Health Promotion and Maintenance Nursing Process:
Evaluation Learning Outcome: 6. Plan evidence-based care for an individual with conjunctivitis and
the family in collaboration with other members of the healthcare team.

6) The nurse is teaching a mother to administer ophthalmic ointment to her infant with conjunctivitis.
The nurse determines that learning goals have been met when the mother performs which action?
A) Places the ointment on a swab and spreads it across closed eyelids.
B) Instills the medication to the lacrimal duct.
C) Uses sterile gauze to apply the ointment.
D) Applies the ointment directly to the conjunctival sac.
Answer: D
Explanation: A) The ointment is applied to the conjunctival sac, not the lids or lacrimal duct, and is
applied directly from the medication tube or vial. Page Ref: 569 Cognitive Level: Analyzing Client
Need: Health Promotion and Maintenance Nursing Process: Evaluation Learning Outcome: 7.
Evaluate expected outcomes for an individual with conjunctivitis.

7) The nurse is providing care for a pediatric client with bacterial conjunctivitis. Which interventions
should the nurse use as part of the collaborative management of the client?
Select all that apply.
A) Dark sunglasses
B) Cold eye compresses
C) Contact the client's school nurse.
D) Careful hand hygiene
E) Antiviral therapy
Answer: A, C, D
Explanation: A) Dark glasses will help to reduce the photophobia that many clients with
conjunctivitis experience. It is often appropriate for the nurse to contact the client's school nurse to
discuss increased prevention and student education. A warm compress, not cold, should be used in
the management of conjunctivitis. Warm compresses relieve discomfort and reduce swelling by
increasing circulation to the eye. Careful hand hygiene is a standard method for managing the client
with conjunctivitis. Antibiotics, not antiviral medications, are prescribed with conjunctivitis.
Page Ref: 569 Cognitive Level: Applying Client Need: Safe and Effective Care Environment Nursing
Process: Implementation Learning Outcome: 5. Summarize therapies used by interdisciplinary
teams in the collaborative care of an individual with conjunctivitis.

8) The nurse is providing discharge instructions for a client who has acute conjunctivitis from
Staphylococcus. Which should the nurse include when teaching this client?
Select all that apply.
A) "It is ok to rub your eyes with a clean, soft cloth for itching."
B) "Do not share towels, make-up, or contact lenses with anyone else, as this can spread the
infection."
C) "You can soak your eyelids with warm saline to soften crusts and exudates that may form."
D) "Wash your hands before cleansing the eye and administering eye drops."
E) "You may go back to sharing towels when the infection is gone."
Answer: B, C, D
Explanation: A) Sharing supplies, such as towels, make-up, or contact lenses, is never a good idea,
due to potential for cross-contamination from person to person. But during an actual infection,
instructions should include discouragement of using the same equipment after the infection is
cleared. Towels need to be washed in hot water. Make-up and contact lenses should be discarded and
not reused. Therefore, this action should be included in instructions. Prevention of cross-
contamination by handwashing (hand hygiene) will minimize the risk of bringing in other organisms
to an already infected eye. This action should be encouraged at all times, but especially when there
already is an infection present. Soaking the lids with sterile saline will soften the crusts from exudates
that accompany the Staphylococcus infection and should be included in the instructions. The action of
rubbing one's eyes can traumatize the eyes further and should not be encouraged. Rubbing also brings
the risk of cross-contamination from other sources, including the other eye if only one eye is involved.
This action is inappropriate and should not be included in care. Page Ref: 569 Cognitive Level:
Applying Client Need: Health Promotion and Maintenance Nursing Process: Implementation
Learning Outcome: 6. Plan evidence-based care for an individual with conjunctivitis and the family in
collaboration with other members of the healthcare team.

9) A nurse is caring for a client diagnosed with trachoma. When providing client education regarding
this condition, the nurse will include which statements?
Select all that apply.
A) "This condition is caused by Chlamydia trachomatis."
B) "It is common in the United States."
C) "It is the primary cause of preventable blindness worldwide."
D) "Early symptoms include inflammation of the cornea."
E) "The causative organism is usually Staphylococcus aureus."
Answer: A, C
Explanation: A) Early manifestations of trachoma include redness, eyelid edema, tearing, and
photophobia. Small conjunctival follicles develop on the upper lids. The inflammation also causes
superficial corneal vascularization and infiltration with granulation tissue. Scarring of the
conjunctival lining of the lid causes entropion (inversion of the eyelid). The lashes then abrade the
cornea, eventually causing ulceration and scarring. The opacity of the scarred cornea results in loss of
vision. This condition is caused by Chlamydia trachomatis, and is endemic in poor, undeveloped
countries, especially where there are water shortages, numerous flies, and crowded living conditions.
Trachoma is rarely seen in the United States except in Native Americans who live in the Southwest.
Page Ref: 567 Cognitive Level: Applying Client Need: Physiological Integrity Nursing Process:
Implementation Learning Outcome: 3. Illustrate the nursing process in providing culturally
competent care across the life span for individuals with conjunctivitis.
10) A nurse is caring for a pediatric client with acute conjunctivitis. (See image.) The client presents
with purulent discharge and crusting of the eye. The nurse suspects that the cause of the condition is:

A) Viral.
B) Allergic.
C) Bacterial.
D) Fungal.
Answer: C
Explanation: A) The major difference between bacterial and viral conjunctivitis is that bacterial
conjunctivitis has a purulent discharge that may result in crusting, whereas the discharge from viral
conjunctivitis is serous (watery). Allergic conjunctivitis produces watery to thick drainage and is
characterized by itching. Fungal is incorrect. Page Ref: 566 Cognitive Level: Applying Client Need:
Physiological Integrity Nursing Process: Assessment Learning Outcome: 1. Describe the
pathophysiology, etiology, clinical manifestations, and direct and indirect causes of conjunctivitis.
Exemplar 9.3 Influenza

1) The nurse is assessing a client with acute malaise and muscle aches. Which questions should the
nurse ask to determine whether the client is experiencing influenza?
Select all that apply.
A) "Have you had a flu shot this year?"
B) "Is your cough productive?"
C) "Have you been exposed to anyone with the flu?"
D) "Are you having any trouble urinating?"
E) "Do you have dizziness?"
Answer: A, B, C
Explanation: A) Based on the presenting symptoms, the nurse would ask whether the client has had a
flu shot or been exposed to the flu. Usually, the cough of an influenza client is nonproductive. A
productive cough may indicate a different diagnosis. Insufficient voiding and dizziness are not routine
manifestations of influenza.
B) Based on the presenting symptoms, the nurse would ask whether the client has had a flu shot or
been exposed to the flu. Usually, the cough of an influenza client is nonproductive. A productive
cough may indicate a different diagnosis. Insufficient voiding and dizziness are not routine
manifestations of influenza.
C) Based on the presenting symptoms, the nurse would ask whether the client has had a flu shot or
been exposed to the flu. Usually, the cough of an influenza client is nonproductive. A productive
cough may indicate a different diagnosis. Insufficient voiding and dizziness are not routine
manifestations of influenza.
D) Based on the presenting symptoms, the nurse would ask whether the client has had a flu shot or
been exposed to the flu. Usually, the cough of an influenza client is nonproductive. A productive
cough may indicate a different diagnosis. Insufficient voiding and dizziness are not routine
manifestations of influenza.
E) Based on the presenting symptoms, the nurse would ask whether the client has had a flu shot or
been exposed to the flu. Usually, the cough of an influenza client is nonproductive. A productive
cough may indicate a different diagnosis. Insufficient voiding and dizziness are not routine
manifestations of influenza.
Page Ref: 570
Cognitive Level: Applying
Client Need: Safe and Effective Care Environment
Nursing Process: Assessment
Learning Outcome: 1. Describe the pathophysiology, etiology, clinical manifestations, and direct and
indirect causes of influenza.
2) The nurse is preparing an educational session on the importance of high-risk populations receiving
an annual influenza vaccination. Which clients are considered high-risk for developing complications
from the flu?
Select all that apply.
A) A 25-year-old pregnant woman at 20 weeks' gestation
B) A 65-year-old woman
C) A 3-year-old with cystic fibrosis
D) A 35-year-old man with a severe allergy to eggs
E) A 20-year-old healthcare worker
Answer: A, B, C, E
Explanation: A) Pregnant women, particularly during the second and third trimesters, are at
increased risk of complications. People at increased risk of influenza or its complications include
infants, young children, and anyone age 50 or older. Patients with chronic disorders, especially
diabetes and cardiac, renal, or pulmonary diseases, are more susceptible as well. The annual flu
vaccination is recommended for all healthcare workers. A 35-year-old man with a severe allergy to
eggs should not get a flu shot, because the vaccine contains eggs and it is not recommended.
B) Pregnant women, particularly during the second and third trimesters, are at increased risk of
complications. People at increased risk of influenza or its complications include infants, young
children, and anyone age 50 or older. Patients with chronic disorders, especially diabetes and cardiac,
renal, or pulmonary diseases, are more susceptible as well. The annual flu vaccination is
recommended for all healthcare workers. A 35-year-old man with a severe allergy to eggs should not
get a flu shot, because the vaccine contains eggs and it is not recommended.
C) Pregnant women, particularly during the second and third trimesters, are at increased risk of
complications. People at increased risk of influenza or its complications include infants, young
children, and anyone age 50 or older. Patients with chronic disorders, especially diabetes and cardiac,
renal, or pulmonary diseases, are more susceptible as well. The annual flu vaccination is
recommended for all healthcare workers. A 35-year-old man with a severe allergy to eggs should not
get a flu shot, because the vaccine contains eggs and it is not recommended.
D) Pregnant women, particularly during the second and third trimesters, are at increased risk of
complications. People at increased risk of influenza or its complications include infants, young
children, and anyone age 50 or older. Patients with chronic disorders, especially diabetes and cardiac,
renal, or pulmonary diseases, are more susceptible as well. The annual flu vaccination is
recommended for all healthcare workers. A 35-year-old man with a severe allergy to eggs should not
get a flu shot, because the vaccine contains eggs and it is not recommended.
E) Pregnant women, particularly during the second and third trimesters, are at increased risk of
complications. People at increased risk of influenza or its complications include infants, young
children, and anyone age 50 or older. Patients with chronic disorders, especially diabetes and cardiac,
renal, or pulmonary diseases, are more susceptible as well. The annual flu vaccination is
recommended for all healthcare workers. A 35-year-old man with a severe allergy to eggs should not
get a flu shot, because the vaccine contains eggs and it is not recommended.
Page Ref: 571
Cognitive Level: Analyzing
Client Need: Health Promotion and Maintenance
Nursing Process: Planning
Learning Outcome: 2. Identify risk factors and prevention methods associated with influenza.
3) The school nurse is planning a teaching session with the parents of students to reduce the spread of
the flu virus throughout the school. What should the nurse include when teaching the parents of a
diverse population about infection-control techniques?
Select all that apply.
A) "Cover your cough" education
B) Appropriate hand hygiene
C) Safe food preparation and storage
D) Sanitizing high-touch items to kill pathogens
E) Withholding immunizations for children with compromised immune systems
Answer: A, B, D
Explanation: A) Sanitizing high-touch items such as toys and all contact surfaces, teaching children
to wash their hands, and appropriate respiratory etiquette such as "cover your cough" education all
control the growth and spread of microorganisms. Teaching parents safe food preparation and storage
is another tool to prevent the spread of microorganisms, but is not related to the flu virus.
Immunizations should not be withheld from immunocompromised children, and this is not an
infection-control strategy. To prevent the spread of communicable diseases, microorganisms must be
killed or their growth controlled.
B) Sanitizing high-touch items such as toys and all contact surfaces, teaching children to wash their
hands, and appropriate respiratory etiquette such as "cover your cough" education all control the
growth and spread of microorganisms. Teaching parents safe food preparation and storage is another
tool to prevent the spread of microorganisms, but is not related to the flu virus. Immunizations should
not be withheld from immunocompromised children, and this is not an infection-control strategy. To
prevent the spread of communicable diseases, microorganisms must be killed or their growth
controlled.
C) Sanitizing high-touch items such as toys and all contact surfaces, teaching children to wash their
hands, and appropriate respiratory etiquette such as "cover your cough" education all control the
growth and spread of microorganisms. Teaching parents safe food preparation and storage is another
tool to prevent the spread of microorganisms, but is not related to the flu virus. Immunizations should
not be withheld from immunocompromised children, and this is not an infection-control strategy. To
prevent the spread of communicable diseases, microorganisms must be killed or their growth
controlled.
D) Sanitizing high-touch items such as toys and all contact surfaces, teaching children to wash their
hands, and appropriate respiratory etiquette such as "cover your cough" education all control the
growth and spread of microorganisms. Teaching parents safe food preparation and storage is another
tool to prevent the spread of microorganisms, but is not related to the flu virus. Immunizations should
not be withheld from immunocompromised children, and this is not an infection-control strategy. To
prevent the spread of communicable diseases, microorganisms must be killed or their growth
controlled.
E) Sanitizing high-touch items such as toys and all contact surfaces, teaching children to wash their
hands, and appropriate respiratory etiquette such as "cover your cough" education all control the
growth and spread of microorganisms. Teaching parents safe food preparation and storage is another
tool to prevent the spread of microorganisms, but is not related to the flu virus. Immunizations should
not be withheld from immunocompromised children, and this is not an infection-control strategy. To
prevent the spread of communicable diseases, microorganisms must be killed or their growth
controlled.
Page Ref: 571
Cognitive Level: Applying
Client Need: Safe and Effective Care Environment
Nursing Process: Implementation
Learning Outcome: 3. Illustrate the nursing process in providing culturally competent care across the
life span for individuals with influenza.

4) A client with the flu is experiencing tachypnea. What intervention should the nurse use to address
the diagnosis of Ineffective Breathing Pattern related to the flu?
Select all that apply.
A) Maintain adequate hydration.
B) Teach the client coughing, deep breathing, and hydration.
C) Prepare the client for the possibility of a tracheostomy tube.
D) Keep the head of the bed elevated.
Answer: A, B, D
Explanation: A) Keeping the head of the bed elevated improves lung excursion and reduces the work
of breathing. Coughing, deep breathing, and, hydration are essential for achieving airway clearance.
Insertion of a tracheostomy and oxygen are not primary treatments for ineffective airway clearance.
B) Keeping the head of the bed elevated improves lung excursion and reduces the work of breathing.
Coughing, deep breathing, and, hydration are essential for achieving airway clearance. Insertion of a
tracheostomy and oxygen are not primary treatments for ineffective airway clearance.
C) Keeping the head of the bed elevated improves lung excursion and reduces the work of breathing.
Coughing, deep breathing, and, hydration are essential for achieving airway clearance. Insertion of a
tracheostomy and oxygen are not primary treatments for ineffective airway clearance.
D) Keeping the head of the bed elevated improves lung excursion and reduces the work of breathing.
Coughing, deep breathing, and, hydration are essential for achieving airway clearance. Insertion of a
tracheostomy and oxygen are not primary treatments for ineffective airway clearance.
Page Ref: 573
Cognitive Level: Analyzing
Client Need: Physiological Integrity
Nursing Process: Implementation
Learning Outcome: 4. Formulate priority nursing diagnoses appropriate for an individual with
influenza.
5) The nurse makes a home visit to a client recovering from complications related to influenza. Which
client statements indicate that desired outcomes have been met?
Select all that apply.
A) "I'm eating healthy foods now."
B) "I went back to work."
C) "I haven't had chills since I left the hospital."
D) "I slept the whole night without coughing."
E) "I was able to take a walk today."
Answer: C, D
Explanation: A) The client who is sleeping at night and is no longer experiencing chills from fever has
recovered and is getting appropriate sleep time. Outcomes for those with the flu do not usually
include walking, going to work, or eating a healthy diet. These would be included in a wellness plan of
care.
B) The client who is sleeping at night and is no longer experiencing chills from fever has recovered
and is getting appropriate sleep time. Outcomes for those with the flu do not usually include walking,
going to work, or eating a healthy diet. These would be included in a wellness plan of care.
C) The client who is sleeping at night and is no longer experiencing chills from fever has recovered
and is getting appropriate sleep time. Outcomes for those with the flu do not usually include walking,
going to work, or eating a healthy diet. These would be included in a wellness plan of care.
D) The client who is sleeping at night and is no longer experiencing chills from fever has recovered
and is getting appropriate sleep time. Outcomes for those with the flu do not usually include walking,
going to work, or eating a healthy diet. These would be included in a wellness plan of care.
E) The client who is sleeping at night and is no longer experiencing chills from fever has recovered
and is getting appropriate sleep time. Outcomes for those with the flu do not usually include walking,
going to work, or eating a healthy diet. These would be included in a wellness plan of care.
Page Ref: 575
Cognitive Level: Analyzing
Client Need: Physiological Integrity
Nursing Process: Evaluation
Learning Outcome: 7. Evaluate expected outcomes for an individual with influenza.
6) The nurse is reviewing diagnostic and laboratory studies performed for an older client with
influenza. Which result should the nurse recognize as being consistent with influenza?
A) Decreased white blood cell count
B) Increased BUN
C) Decreased sodium level
D) Fluid-filled lungs on chest x-ray
Answer: A
Explanation: A) The white blood cell count of a client with influenza will typically be decreased.
Laboratory tests for BUN and sodium levels are not usually associated with influenza. Unless the
client with influenza develops complications, the chest x-ray is clear.
B) The white blood cell count of a client with influenza will typically be decreased. Laboratory tests for
BUN and sodium levels are not usually associated with influenza. Unless the client with influenza
develops complications, the chest x-ray is clear.
C) The white blood cell count of a client with influenza will typically be decreased. Laboratory tests for
BUN and sodium levels are not usually associated with influenza. Unless the client with influenza
develops complications, the chest x-ray is clear.
D) The white blood cell count of a client with influenza will typically be decreased. Laboratory tests for
BUN and sodium levels are not usually associated with influenza. Unless the client with influenza
develops complications, the chest x-ray is clear.
Page Ref: 572
Cognitive Level: Analyzing
Client Need: Physiological Integrity
Nursing Process: Assessment
Learning Outcome: 5. Summarize therapies used by interdisciplinary teams in the collaborative care
of an individual with influenza.
7) The nurse is planning care for a client diagnosed with influenza. Which interventions should the
nurse include when planning this client's care?
Select all that apply.
A) Placing droplet and contact precaution signs on the client room door
B) Placing the client in a negative air flow room
C) Placing a ventilator in the room
D) Notifying other departments of the diagnosis
E) Using appropriate PPE
Answer: A, E
Explanation: A) To prevent the spread of influenza, the client is placed in a private room with signs
for droplet and contact precautions. It is appropriate for the healthcare workers to use appropriate
PPE for these transmission-based precautions. Placing signs on the door is the way to notify other
departments of precautions. Negative air flow rooms are for diseases such as chicken pox, measles,
and SARS. There is no indication that this client will need a ventilator.
B) To prevent the spread of influenza, the client is placed in a private room with signs for droplet and
contact precautions. It is appropriate for the healthcare workers to use appropriate PPE for these
transmission-based precautions. Placing signs on the door is the way to notify other departments of
precautions. Negative air flow rooms are for diseases such as chicken pox, measles, and SARS. There
is no indication that this client will need a ventilator.
C) To prevent the spread of influenza, the client is placed in a private room with signs for droplet and
contact precautions. It is appropriate for the healthcare workers to use appropriate PPE for these
transmission-based precautions. Placing signs on the door is the way to notify other departments of
precautions. Negative air flow rooms are for diseases such as chicken pox, measles, and SARS. There
is no indication that this client will need a ventilator.
D) To prevent the spread of influenza, the client is placed in a private room with signs for droplet and
contact precautions. It is appropriate for the healthcare workers to use appropriate PPE for these
transmission-based precautions. Placing signs on the door is the way to notify other departments of
precautions. Negative air flow rooms are for diseases such as chicken pox, measles, and SARS. There
is no indication that this client will need a ventilator.
E) To prevent the spread of influenza, the client is placed in a private room with signs for droplet and
contact precautions. It is appropriate for the healthcare workers to use appropriate PPE for these
transmission-based precautions. Placing signs on the door is the way to notify other departments of
precautions. Negative air flow rooms are for diseases such as chicken pox, measles, and SARS. There
is no indication that this client will need a ventilator.
Page Ref: 574-575
Cognitive Level: Applying
Client Need: Safe and Effective Care Environment
Nursing Process: Implementation
Learning Outcome: 6. Plan evidence-based care for an individual with influenza and the family in
collaboration with other members of the healthcare team.
Exemplar 9.4 Otitis Media

1) The nurse walks into an examination room and sees a young child demonstrate a specific behavior.
Which health problem should the nurse suspect the child is experiencing?

A) Sore throat
B) Hunger
C) Otitis media
D) Head cold
Answer: C
Explanation: A) Pulling at the ear is a characteristic sign of otitis media in a young child. Pulling at
the ear does not indicate a sore throat, hunger, or a head cold. Page Ref: 578 Cognitive Level:
Analyzing Client Need: Physiological Integrity Nursing Process: Assessment Learning Outcome: 1.
Describe the pathophysiology, etiology, clinical manifestations, and direct and indirect causes of otitis
media.

2) A client with newly diagnosed otitis media tells the nurse that the left ear has been aching for
weeks. Since this health problem has been untreated for so long, which additional problem is this
client at risk for developing?
A) External otitis
B) Meningitis
C) Pneumonia
D) Influenza
Answer: B
Explanation: A) The bacterial infection from otitis media may also migrate internally, leading to the
development of bacterial meningitis. Otitis media is not known to cause external otitis. Otitis media
does not cause pneumonia or influenza. Page Ref: 576 Cognitive Level: Analyzing Client Need: Safe
and Effective Care Environment Nursing Process: Assessment Learning Outcome: 2. Identify risk
factors and prevention methods associated with otitis media.
3) A new mother is distraught because her infant has a fever of 102ºF and is diagnosed with otitis
media. What should the nurse instruct the mother to help the child's fever and pain?
A) Swaddle the baby in blankets.
B) Feed the baby solid foods.
C) Administer acetaminophen.
D) Bathe the baby with cool water.
Answer: C
Explanation: A) Swaddling the baby with blankets is not going to reduce the fever or help the pain.
The baby may not be of the age to take solid foods. Acetaminophen will help reduce the child's fever
and reduce the pain. Bathing with cool water is not an appropriate intervention to reduce the fever of
a baby. Page Ref: 582 Cognitive Level: Applying Client Need: Physiological Integrity Nursing
Process: Implementation Learning Outcome: 3. Illustrate the nursing process in providing culturally
competent care across life span for individuals with otitis media.

4) An adolescent with otitis media is experiencing extreme pain. Which should the nurse emphasize to
address the diagnosis of Acute Pain for this client?
A) Apply a cold compress to the affected ear.
B) Report abrupt relief of pain immediately.
C) Continue plans for air travel.
D) Report increased pain when moving the outer ear.
Answer: B
Explanation: A) Abrupt relief of pain may mean that the tympanic membrane has perforated. Heat
should be applied to dilate surrounding blood vessels and decrease swelling. The pain of otitis media
is not aggravated by movement of the external ear. Drastic changes in barometric pressure can
increase pain considerably, so clients are discouraged from traveling by air. Page Ref: 582 Cognitive
Level: Applying Client Need: Physiological Integrity Nursing Process: Implementation Learning
Outcome: 4. Formulate priority nursing diagnoses appropriate for an individual with otitis media.

5) During a home visit, the nurse instructs a young mother to bottle-feed the baby in the upright
position. Which health problem will this position help prevent from developing in this child?
A) Choking
B) Aspiration
C) Sinus infection
D) Otitis media
Answer: D
Explanation: A) Infants and small children who are bottle-fed in a supine position have a greater
probability of developing otitis media because the eustachian tube opens when the child sucks, and
the horizontal angle provides easy access to the middle ear. Children are not prone to sinus infection
at that age. Choking results from holes in bottle nipples that are too big. Aspiration is caused by
feeding too much too fast. Page Ref: 577 Cognitive Level: Applying Client Need: Health Promotion
and Maintenance Nursing Process: Implementation Learning Outcome: 2. Identify risk factors and
prevention methods associated with otitis media.

6) A client has completed the full course of antibiotics prescribed to treat otitis media. Which primary
manifestation of the disorder will be relieved as evidence that treatment has been effective?
A) Impaired hearing
B) Dizziness
C) Pain
D) Nausea and vomiting
Answer: C
Explanation: A) Ear pain is the most common symptom of otitis media that motivates the client to
seek health care. Secondary symptoms associated with the disease include dizziness, impaired
hearing, and nausea and vomiting. Page Ref: 582 Cognitive Level: Analyzing Client Need:
Physiological Integrity Nursing Process: Evaluation Learning Outcome: 7. Evaluate expected
outcomes for an individual with otitis media.

7) A 2-year-old child with otitis media is prescribed amoxicillin clavulanate 250 mg/5 mL three times
daily by mouth for 10 days. What should the nurse teach the mother about this medication?
A) "It is ok to stop the antibiotic if the child begins to have side effects."
B) "Give the antibiotic for the full 10 days as prescribed."
C) "It is important to measure the prescribed dose in a household teaspoon."
D) "Be sure to administer a loading dose of the medication when you get home."
Answer: B
Explanation: A) Antibiotics must be administered for the full number of days ordered to prevent
mutation of resistant strains of bacteria. A loading dose was not ordered. A household teaspoon could
contain less than 5ml, and the full dose must be given. Stopping the antibiotic before the prescribed
time will permit remaining bacteria to reproduce, and the otitis media will return, possibly with
antibiotic-resistant organisms. Page Ref: 579 Cognitive Level: Applying Client Need: Physiological
Integrity Nursing Process: Implementation Learning Outcome: 5. Summarize therapies used by
interdisciplinary teams in the collaborative care of an individual with otitis media.

8) The nurse is teaching the parents of an infant with acute otitis media. What would be most
important for the nurse to teach the parents?
A) Administer a decongestant for nasal congestion.
B) Keep the baby in a flat position during sleep.
C) Administer acetaminophen to relieve pain and decrease fever.
D) Place the baby to sleep with a pacifier.
Answer: C
Explanation: A) Parents are taught to administer acetaminophen to relieve the discomfort and
decrease fever associated with acute otitis media. Decongestants are not recommended for treatment
of acute otitis media. A flat position could exacerbate the discomfort. Elevating the head slightly is
recommended. Placing infants to sleep with a pacifier can increase the incidence of otitis media.
Page Ref: 580 Cognitive Level: Applying Client Need: Physiological Integrity Nursing Process:
Implementation Learning Outcome: 6. Plan evidence-based care for an individual with otitis media
and the family in collaboration with other members of the healthcare team.
Exemplar 9.5 Pneumonia

1) An 86-year-old client is admitted with pneumonia. What manifestations would the nurse expect to
find when assessing this client?
Select all that apply.
A) Hemoptysis
B) Increased appetite
C) Change in level of consciousness
D) Respiration of 24
E) Lethargy
Answer: A, C, D, E
Explanation: A) Frequently, caregivers or family members note that the client looks generally ill. The
client is lethargic and less coherent, and has stopped eating and drinking. Hemoptysis is seen in
pneumonia, and the respiratory rate would be greater than 20. Page Ref: 587 Cognitive Level:
Applying Client Need: Physiological Integrity Nursing Process: Assessment Learning Outcome: 1.
Describe the pathophysiology, etiology, and clinical manifestations of pneumonia.

2) A nurse is caring for a 72-year-old male client admitted to the hospital with pneumonia. The client
asks the nurse if there are things he can do to decrease the risk for developing pneumonia in the
future. Which would be the most appropriate response by the nurse?
Select all that apply.
A) "You should drink a yogurt drink once a day that is supplemented with L. casei immunitas
cultures."
B) "You can start by not smoking."
C) "You can get the pneumonia vaccination, which may help to decrease your risk in the future."
D) "Avoiding alcohol will help."
E) "There is nothing that you can do to decrease your risk of pneumonia in the future."
Answer: B, C, D
Explanation: A) Research indicates a high rate of pneumonia in clients with frequent exposure to
cigarette smoke and alcohol use. Smoking injures tissues in the airways and decreases the action of
cilia. Chemicals in cigarettes have a numbing effect on the cough reflex. Alcohol interferes with the
actions of macrophages. Studies suggest that supplements containing micronutrients that
approximate the recommended dietary allowances can assist in preventing infections. A balanced diet
can also decrease the risk of respiratory infections. Pneumonia vaccines can also be considered to
decrease the risk of development in the future. There is not an established body of scientific evidence
that supports the claim that L. casei immunitas cultures can improve immune function. Telling the
client he cannot decrease his risk in the future is incorrect. Page Ref: 588 Cognitive Level: Applying
Client Need: Health Promotion and Maintenance Nursing Process: Implementation Learning
Outcome: 2. Identify risk factors and prevention methods associated with pneumonia.

3) The nurse is caring for a 72-year-old client who is hospitalized with a second episode of pneumonia
in the past 18 months. The client has expressed frustration to the nurse and states, "I never got sick
when I was younger. Why is this happening?" Which response by the nurse is most appropriate?
A) "As you grow older, your immune system just quits working."
B) "As you grow older, there is a decrease in the immune response, which puts you at greater risk for
developing an infection."
C) "As you grow older, there in an overall increase in the speed and strength of your immune
response."
D) "As you grow older, there is an increase in the number of B cells in the circulation, which hinders
the immune response."
Answer: B
Explanation: A) As a person grows older, there is an overall decrease in the speed and strength of the
immune response. The immune system does not quit working totally. There is a decrease in the
number of B cells in circulation. Page Ref: 590 Cognitive Level: Applying Client Need: Health
Promotion and Maintenance Nursing Process: Implementation Learning Outcome: 3. Illustrate the
nursing process in providing culturally competent care across the life span for individuals with
pneumonia.

4) The nurse determines that the diagnosis of Ineffective Airway Clearance is appropriate for a client
with pneumonia who is experiencing copious amounts of respiratory secretions. Which intervention
should the nurse include in this client's plan of care?
A) Perform chest percussion every 4 hours and prn.
B) Administer the pneumococcal vaccine prior to discharge.
C) Limit fluid intake to 1,000 mL per day.
D) Provide the client with smoking cessation education.
Answer: A
Explanation: A) Chest percussion can help clear secretions. Providing education for smoking
cessation and administering the pneumococcal vaccine are important in treating a client with
pneumonia; however, they would be aligned with a different nursing diagnosis. Patients with
pneumonia are encouraged to increase fluid intake. Page Ref: 596 Cognitive Level: Applying
Client Need: Physiological Integrity Nursing Process: Planning Learning Outcome: 4. Formulate
priority nursing diagnoses appropriate for an individual with pneumonia.

5) The nurse is providing discharge teaching to a client recovering from pneumonia. Which client
statement indicates that additional teaching is needed?
A) "I can't get the influenza vaccine due to my allergy to eggs."
B) "I will get the influenza vaccine every year."
C) "I will get the pneumococcal vaccine every fall."
D) "I will get the pneumococcal vaccine as soon as I recover from this pneumonia."
Answer: C
Explanation: A) Influenza vaccine is administered annually to healthy individuals and should not be
given to those with an allergy to eggs. The pneumococcal vaccine is administered once. Revaccination
is only recommended in persons with renal failure, those who have had splenectomies, those with
malignancies, and those with HIV/AIDS. Page Ref: 597 Cognitive Level: Analyzing Client Need:
Health Promotion and Maintenance Nursing Process: Evaluation Learning Outcome: 7. Evaluate
expected outcomes for an individual with pneumonia.

6) A client with pneumonia develops a fever. Which interventions should the nurse use to attain the
goal of normal body temperature?
Select all that apply.
A) Increase the temperature of the room environment to prevent shivering.
B) Administer antipyretic medications.
C) Restrict fluids during periods of hyperthermia because of the risk of electrolyte imbalance.
D) Use ice packs and a tepid bath every 2 hours.
E) Promote frequent rest periods to increase energy reserve.
Answer: B, E
Explanation: A) Hyperthermia is an expected consequence of the infectious disease process. Fever
can produce mild, short-term effects and, when prolonged, can cause life-threatening effects. The
nurse should administer antipyretic medications as indicated for elevated temperatures and enforce
frequent rest periods because rest increases energy reserve that is depleted by increased metabolic,
heart, and respiratory rates. The nurse should use ice packs, cool/tepid baths, or a hypothermia
blanket with caution and only as needed. The nurse should encourage fluid intake rather than restrict
fluids because of the risk of electrolyte imbalance. Page Ref: 595 Cognitive Level: Applying Client
Need: Physiological Integrity Nursing Process: Implementation Learning Outcome: 6. Plan
evidence-based care for an individual with pneumonia and the family in collaboration with other
members of the healthcare team.

7) The nurse is caring for a client who develops a fever and productive cough after having an
appendectomy. Which orders should the nurse expect from the healthcare provider for this health
problem?
Select all that apply.
A) Sputum cultures
B) Antibiotics
C) Chest physiotherapy
D) Bronchial washing for culture
E) Isolation precautions
Answer: A, B, C
Explanation: A) The nurse would expect to obtain sputum cultures, administer antibiotics, and
perform chest physiotherapy to help clear the respiratory secretions. Bronchial washings are not
routine testing for this scenario. The client likely has an infectious disease and is not contagious.
Isolation precautions are usually not ordered for noncontagious infections. Page Ref: 595 Cognitive
Level: Analyzing Client Need: Physiological Integrity Nursing Process: Planning Learning Outcome:
5. Summarize therapies used by interdisciplinary teams in the collaborative care of an individual with
pneumonia.
Exemplar 9.6 Sepsis

1) A client is admitted to the Intensive Care Unit with a systemic infection. What manifestations will
the nurse most likely assess in this client?
Select all that apply.
A) Tachycardia
B) Pain
C) Edema
D) Anorexia
E) Fever
Answer: A, D, E
Explanation: A) Fever, tachycardia, and anorexia are the most common symptoms of a systemic
infection. Edema and pain are symptoms of a local infection.
B) Fever, tachycardia, and anorexia are the most common symptoms of a systemic infection. Edema
and pain are symptoms of a local infection.
C) Fever, tachycardia, and anorexia are the most common symptoms of a systemic infection. Edema
and pain are symptoms of a local infection.
D) Fever, tachycardia, and anorexia are the most common symptoms of a systemic infection. Edema
and pain are symptoms of a local infection.
E) Fever, tachycardia, and anorexia are the most common symptoms of a systemic infection. Edema
and pain are symptoms of a local infection.
Page Ref: 601
Cognitive Level: Analyzing
Client Need: Physiological Integrity
Nursing Process: Assessment
Learning Outcome: 1. Describe the pathophysiology, etiology, clinical manifestations, and direct and
indirect causes of sepsis.
2) The nurse is preparing an educational session on sepsis. What should the nurse include as a major
risk factor for the development of this health problem?
A) Pneumococcal bacteria
B) Leukocytosis on the complete blood count
C) Undiagnosed urinary tract infection
D) Elevated temperature
Answer: C
Explanation: A) Sepsis is an entire-body inflammatory process. Sepsis is most often the result of
gram-positive infections from Staphylococcus and Streptococcus bacteria but may also follow gram-
negative bacterial infections such as Pseudomonas, Escherichia coli, and Klebsiella. A portal of entry
for sepsis is the urinary system. Leukocytosis occurs with sepsis if the client is able to mount an
immune response. An elevated temperature is a manifestation of sepsis.
B) Sepsis is an entire-body inflammatory process. Sepsis is most often the result of gram-positive
infections from Staphylococcus and Streptococcus bacteria but may also follow gram-negative
bacterial infections such as Pseudomonas, Escherichia coli, and Klebsiella. A portal of entry for sepsis
is the urinary system. Leukocytosis occurs with sepsis if the client is able to mount an immune
response. An elevated temperature is a manifestation of sepsis.
C) Sepsis is an entire-body inflammatory process. Sepsis is most often the result of gram-positive
infections from Staphylococcus and Streptococcus bacteria but may also follow gram-negative
bacterial infections such as Pseudomonas, Escherichia coli, and Klebsiella. A portal of entry for sepsis
is the urinary system. Leukocytosis occurs with sepsis if the client is able to mount an immune
response. An elevated temperature is a manifestation of sepsis.
D) Sepsis is an entire-body inflammatory process. Sepsis is most often the result of gram-positive
infections from Staphylococcus and Streptococcus bacteria but may also follow gram-negative
bacterial infections such as Pseudomonas, Escherichia coli, and Klebsiella. A portal of entry for sepsis
is the urinary system. Leukocytosis occurs with sepsis if the client is able to mount an immune
response. An elevated temperature is a manifestation of sepsis.
Page Ref: 601
Cognitive Level: Applying
Client Need: Physiological Integrity
Nursing Process: Implementation
Learning Outcome: 2. Identify risk factors and prevention methods associated with sepsis.
3) A 1-month-old infant is admitted to the hospital with a temperature of 102°F. Why is this client
going to be evaluated for the presence of sepsis?
A) Absence of sweat glands
B) Immature immune system
C) Inadequate red blood cells
D) Poor lung elasticity
Answer: B
Explanation: A) A child less than 3 months of age with a temperature higher than 100.4°F should be
evaluated for sepsis because the child is at increased risk secondary to an immature immune system.
The child is not evaluated for sepsis because of the absence of sweat glands, inadequate red blood
cells, or poor lung elasticity.
B) A child less than 3 months of age with a temperature higher than 100.4°F should be evaluated for
sepsis because the child is at increased risk secondary to an immature immune system. The child is
not evaluated for sepsis because of the absence of sweat glands, inadequate red blood cells, or poor
lung elasticity.
C) A child less than 3 months of age with a temperature higher than 100.4°F should be evaluated for
sepsis because the child is at increased risk secondary to an immature immune system. The child is
not evaluated for sepsis because of the absence of sweat glands, inadequate red blood cells, or poor
lung elasticity.
D) A child less than 3 months of age with a temperature higher than 100.4°F should be evaluated for
sepsis because the child is at increased risk secondary to an immature immune system. The child is
not evaluated for sepsis because of the absence of sweat glands, inadequate red blood cells, or poor
lung elasticity.
Page Ref: 601
Cognitive Level: Analyzing
Client Need: Physiological Integrity
Nursing Process: Assessment
Learning Outcome: 3. Illustrate the nursing process in providing culturally competent care across the
life span for individuals with sepsis.
4) The nurse identifies the diagnosis of Ineffective Peripheral Tissue Perfusion as being appropriate
for a client with septicemia. Which intervention will address this client's health problem?
A) Monitor heart rate every hour.
B) Assess temperature every 4 hours.
C) Monitor pupil reactions every 8 hours.
D) Monitor for cyanosis.
Answer: D
Explanation: A) Assessing temperature and monitoring heart rate and pupil reaction are important
when assessing a client with septicemia, but a change in skin color will alert the nurse immediately of
decreased tissue perfusion.
B) Assessing temperature and monitoring heart rate and pupil reaction are important when assessing
a client with septicemia, but a change in skin color will alert the nurse immediately of decreased tissue
perfusion.
C) Assessing temperature and monitoring heart rate and pupil reaction are important when assessing
a client with septicemia, but a change in skin color will alert the nurse immediately of decreased tissue
perfusion.
D) Assessing temperature and monitoring heart rate and pupil reaction are important when assessing
a client with septicemia, but a change in skin color will alert the nurse immediately of decreased tissue
perfusion.
Page Ref: 603
Cognitive Level: Applying
Client Need: Physiological Integrity
Nursing Process: Implementation
Learning Outcome: 4. Formulate priority nursing diagnoses appropriate for an individual with sepsis.
5) An elderly client who resides in a long-term care facility is admitted to the hospital with sepsis. The
family is tearful and does not understand how their family member got so sick from a bed sore. What
should the nurse teach the family to help in the care of the client?
A) Alert the staff when the IV runs dry.
B) Help the nurse with dressing changes.
C) Assist the client to the bathroom so there is not a fall.
D) Assist the client with meals to obtain optimal nourishment.
Answer: D
Explanation: A) Poor nutritional status and a slow-functioning immune system contribute to the risk
for sepsis in the older client. The family can help by assisting and encouraging the client to take in the
proper nourishment needed. The family is not responsible for IV or dressing changes. Assisting the
client to the bathroom should be done by the staff who are responsible for client safety.
B) Poor nutritional status and a slow-functioning immune system contribute to the risk for sepsis in
the older client. The family can help by assisting and encouraging the client to take in the proper
nourishment needed. The family is not responsible for IV or dressing changes. Assisting the client to
the bathroom should be done by the staff who are responsible for client safety.
C) Poor nutritional status and a slow-functioning immune system contribute to the risk for sepsis in
the older client. The family can help by assisting and encouraging the client to take in the proper
nourishment needed. The family is not responsible for IV or dressing changes. Assisting the client to
the bathroom should be done by the staff who are responsible for client safety.
D) Poor nutritional status and a slow-functioning immune system contribute to the risk for sepsis in
the older client. The family can help by assisting and encouraging the client to take in the proper
nourishment needed. The family is not responsible for IV or dressing changes. Assisting the client to
the bathroom should be done by the staff who are responsible for client safety.
Page Ref: 528
Cognitive Level: Applying
Client Need: Physiological Integrity
Nursing Process: Implementation
Learning Outcome: 6. Plan evidence-based care for an individual with sepsis and the family in
collaboration with other members of the healthcare team.
6) An elderly client with sepsis has been admitted to the nursing unit. The nurse is planning care and
determines that one goal for this client is to maintain normal mental status. Which outcome
evaluation implies that the goal has been met?
A) The client is agitated.
B) The client has a Glasgow coma score of 4.
C) The client responds to questions appropriately.
D) The client's pupils are fixed and dilated.
Answer: C
Explanation: A) A client who responds to questions appropriately is alert and oriented. Restlessness
or agitation in a client with sepsis can be a sign that the client's condition is deteriorating. The client
with a Glasgow coma score of 4 is comatose. When pupils are fixed and do not respond to light, death
is imminent.
B) A client who responds to questions appropriately is alert and oriented. Restlessness or agitation in
a client with sepsis can be a sign that the client's condition is deteriorating. The client with a Glasgow
coma score of 4 is comatose. When pupils are fixed and do not respond to light, death is imminent.
C) A client who responds to questions appropriately is alert and oriented. Restlessness or agitation in
a client with sepsis can be a sign that the client's condition is deteriorating. The client with a Glasgow
coma score of 4 is comatose. When pupils are fixed and do not respond to light, death is imminent.
D) A client who responds to questions appropriately is alert and oriented. Restlessness or agitation in
a client with sepsis can be a sign that the client's condition is deteriorating. The client with a Glasgow
coma score of 4 is comatose. When pupils are fixed and do not respond to light, death is imminent.
Page Ref: 604
Cognitive Level: Analyzing
Client Need: Physiological Integrity
Nursing Process: Evaluation
Learning Outcome: 7. Evaluate expected outcomes for an individual with sepsis.
7) The healthcare provider has prescribed a client to have peak and trough blood levels drawn to
evaluate the therapeutic effect of an IV antibiotic. When should the nurse schedule the blood samples
to be drawn?
Select all that apply.
A) Prior to the discontinuing the antibiotic
B) A few minutes before the next scheduled dose of medication
C) During the infusion of the antibiotic
D) 30 minutes after the IV administration
E) 1-2 hours after the oral administration of the medication
Answer: B, D
Explanation: A) Antibiotic peak and trough levels monitor therapeutic blood levels of the prescribed
medication. The therapeutic range–that is, the minimum and maximum blood levels at which the
drug is effective–is known for a given drug. By measuring blood levels at the predicted peak (1-2
hours after oral administration, 1 hour after intramuscular administration, and 30 minutes after IV
administration) and trough, usually a few minutes before the next scheduled dose, it is also possible
to determine whether the drug is reaching a toxic or harmful level during therapy, increasing the
likelihood of adverse effects.
B) Antibiotic peak and trough levels monitor therapeutic blood levels of the prescribed medication.
The therapeutic range–that is, the minimum and maximum blood levels at which the drug is
effective–is known for a given drug. By measuring blood levels at the predicted peak (1-2 hours after
oral administration, 1 hour after intramuscular administration, and 30 minutes after IV
administration) and trough, usually a few minutes before the next scheduled dose, it is also possible
to determine whether the drug is reaching a toxic or harmful level during therapy, increasing the
likelihood of adverse effects.
C) Antibiotic peak and trough levels monitor therapeutic blood levels of the prescribed medication.
The therapeutic range–that is, the minimum and maximum blood levels at which the drug is
effective–is known for a given drug. By measuring blood levels at the predicted peak (1-2 hours after
oral administration, 1 hour after intramuscular administration, and 30 minutes after IV
administration) and trough, usually a few minutes before the next scheduled dose, it is also possible
to determine whether the drug is reaching a toxic or harmful level during therapy, increasing the
likelihood of adverse effects.
D) Antibiotic peak and trough levels monitor therapeutic blood levels of the prescribed medication.
The therapeutic range–that is, the minimum and maximum blood levels at which the drug is
effective–is known for a given drug. By measuring blood levels at the predicted peak (1-2 hours after
oral administration, 1 hour after intramuscular administration, and 30 minutes after IV
administration) and trough, usually a few minutes before the next scheduled dose, it is also possible
to determine whether the drug is reaching a toxic or harmful level during therapy, increasing the
likelihood of adverse effects.
E) Antibiotic peak and trough levels monitor therapeutic blood levels of the prescribed medication.
The therapeutic range–that is, the minimum and maximum blood levels at which the drug is
effective–is known for a given drug. By measuring blood levels at the predicted peak (1-2 hours after
oral administration, 1 hour after intramuscular administration, and 30 minutes after IV
administration) and trough, usually a few minutes before the next scheduled dose, it is also possible
to determine whether the drug is reaching a toxic or harmful level during therapy, increasing the
likelihood of adverse effects.
Page Ref: 557
Cognitive Level: Applying
Client Need: Physiological Integrity
Nursing Process: Implementation
Learning Outcome: 5. Summarize therapies used by interdisciplinary teams in the collaborative care
of an individual with sepsis.

8) An 80-year-old client is recovering in the ICU from septicemia. Which intervention will help
prevent further infection for this client?
A) Foley drainage on the bed at the client's feet
B) Oral and skin care
C) Turn, cough, and deep breathe q shift.
D) Sterile wound care
Answer: B
Explanation: A) Good oral and skin care will prevent breakdown and prevent entry by bacteria. In
order to prevent skin breakdown and promote respiratory function, the client is turned at least every
2 hours. The Foley drainage bag is always kept below the level of the client's bladder to prevent reflux.
There is no evidence that this client has a wound.
B) Good oral and skin care will prevent breakdown and prevent entry by bacteria. In order to prevent
skin breakdown and promote respiratory function, the client is turned at least every 2 hours. The
Foley drainage bag is always kept below the level of the client's bladder to prevent reflux. There is no
evidence that this client has a wound.
C) Good oral and skin care will prevent breakdown and prevent entry by bacteria. In order to prevent
skin breakdown and promote respiratory function, the client is turned at least every 2 hours. The
Foley drainage bag is always kept below the level of the client's bladder to prevent reflux. There is no
evidence that this client has a wound.
D) Good oral and skin care will prevent breakdown and prevent entry by bacteria. In order to prevent
skin breakdown and promote respiratory function, the client is turned at least every 2 hours. The
Foley drainage bag is always kept below the level of the client's bladder to prevent reflux. There is no
evidence that this client has a wound.
Page Ref: 544
Cognitive Level: Applying
Client Need: Physiological Integrity
Nursing Process: Implementation
Learning Outcome: 2. Identify risk factors and prevention methods associated with sepsis.
9) A nurse working in the ICU is receiving a client diagnosed with early septic shock from the
Emergency Department. The nurse will recognize which symptoms associated with this condition?
Select all that apply.
A) Normal blood pressure
B) Rapid and deep respirations
C) Shallow respirations
D) Warm and flushed skin
E) Lethargic mental status
F) Decreased urine output
Answer: A, B, D
Explanation: A) Septic shock has an early phase and a late phase. Early-phase manifestations include
normal blood pressure, rapid and deep respirations, and warm or flushed skin. The other
manifestations occur in the late phase of shock.
B) Septic shock has an early phase and a late phase. Early-phase manifestations include normal blood
pressure, rapid and deep respirations, and warm or flushed skin. The other manifestations occur in
the late phase of shock.
C) Septic shock has an early phase and a late phase. Early-phase manifestations include normal blood
pressure, rapid and deep respirations, and warm or flushed skin. The other manifestations occur in
the late phase of shock.
D) Septic shock has an early phase and a late phase. Early-phase manifestations include normal blood
pressure, rapid and deep respirations, and warm or flushed skin. The other manifestations occur in
the late phase of shock.
E) Septic shock has an early phase and a late phase. Early-phase manifestations include normal blood
pressure, rapid and deep respirations, and warm or flushed skin. The other manifestations occur in
the late phase of shock.
F) Septic shock has an early phase and a late phase. Early-phase manifestations include normal blood
pressure, rapid and deep respirations, and warm or flushed skin. The other manifestations occur in
the late phase of shock.
Page Ref: 601
Cognitive Level: Applying
Client Need: Physiological Integrity
Nursing Process: Assessment
Learning Outcome: 1. Describe the pathophysiology, etiology, clinical manifestations, and direct and
indirect causes of sepsis.
10) List the pathophysiology concepts related to the onset of sepsis in sequential order.
1. Macrophage-producing cytokines are released.
2. Endotoxin released by microorganisms sets off an out-of-control inflammatory process.
3. Neutrophils arrive and multiply, occluding capillaries.
4. Vasodilation with increased capillary permeability and fluid leak.
Answer: 2, 1, 4, 3
Explanation: The pathophysiology of sepsis occurs as follows: Endotoxin released by microorganisms
sets off an out-of-control inflammatory process; macrophage-producing cytokines are released;
vasodilation with increased capillary permeability and fluid leak; neutrophils arrive and multiply,
occluding capillaries.
Page Ref: 600
Cognitive Level: Analyzing
Client Need: Physiological Integrity
Nursing Process: Planning
Learning Outcome: 1. Describe the pathophysiology, etiology, clinical manifestations, and direct and
indirect causes of sepsis.

Exemplar 9.7 Tuberculosis

1) The nurse is preparing to assess a 90-year-old client admitted with tuberculosis. Which
manifestation will the nurse most likely assess in this client?
A) Night sweats
B) Swollen lymph nodes
C) Cough
D) Hemoptysis
Answer: C
Explanation: A) Presenting symptoms of tuberculosis in the older adult are often vague and include
coughing, weight loss, diminished appetite, and periodic fevers. Night sweats, swollen lymph nodes,
and hemoptysis are not considered presenting symptoms of tuberculosis in the older adult.
B) Presenting symptoms of tuberculosis in the older adult are often vague and include coughing,
weight loss, diminished appetite, and periodic fevers. Night sweats, swollen lymph nodes, and
hemoptysis are not considered presenting symptoms of tuberculosis in the older adult.
C) Presenting symptoms of tuberculosis in the older adult are often vague and include coughing,
weight loss, diminished appetite, and periodic fevers. Night sweats, swollen lymph nodes, and
hemoptysis are not considered presenting symptoms of tuberculosis in the older adult.
D) Presenting symptoms of tuberculosis in the older adult are often vague and include coughing,
weight loss, diminished appetite, and periodic fevers. Night sweats, swollen lymph nodes, and
hemoptysis are not considered presenting symptoms of tuberculosis in the older adult.
Page Ref: 609
Cognitive Level: Applying
Client Need: Physiological Integrity
Nursing Process: Assessment
Learning Outcome: 1. Describe the pathophysiology, etiology, clinical manifestations, and direct and
indirect causes of tuberculosis.
2) A 15-year-old client is brought to the Emergency Department with fatigue, weight loss, a dry cough,
and night sweats. The family just recently immigrated to the United States. For which potential risk
should the nurse plan this client's care?
A) Pneumothorax
B) Pneumonia
C) Renal failure
D) Septicemia
Answer: A
Explanation: A) This client was foreign-born, a risk factor for TB, and has the classic symptoms of
tuberculosis. The nurse plans frequent respiratory assessments, as this child is at risk for
pneumothorax. Patients with TB are not at particular risk for pneumonia, renal failure, or septicemia.
B) This client was foreign-born, a risk factor for TB, and has the classic symptoms of tuberculosis. The
nurse plans frequent respiratory assessments, as this child is at risk for pneumothorax. Patients with
TB are not at particular risk for pneumonia, renal failure, or septicemia.
C) This client was foreign-born, a risk factor for TB, and has the classic symptoms of tuberculosis. The
nurse plans frequent respiratory assessments, as this child is at risk for pneumothorax. Patients with
TB are not at particular risk for pneumonia, renal failure, or septicemia.
D) This client was foreign-born, a risk factor for TB, and has the classic symptoms of tuberculosis. The
nurse plans frequent respiratory assessments, as this child is at risk for pneumothorax. Patients with
TB are not at particular risk for pneumonia, renal failure, or septicemia.
Page Ref: 609
Cognitive Level: Analyzing
Client Need: Physiological Integrity
Nursing Process: Planning
Learning Outcome: 2. Identify risk factors and prevention methods associated with tuberculosis.
3) The nurse in an inner city clinic is providing a health screening for a homeless man with a history
of drug abuse. The client has a chronic non-productive cough. For what should the nurse expect to
screen this client?
A) Herpes zoster
B) Sickle cell disease
C) Sick sinus syndrome
D) Tuberculosis
Answer: D
Explanation: A) The homeless client who abuses drugs is at risk for contracting tuberculosis. The
nurse would expect to screen this client for TB. There is no evidence to support the need to screen the
client for sickle cell disease, herpes zoster, or sick sinus syndrome.
B) The homeless client who abuses drugs is at risk for contracting tuberculosis. The nurse would
expect to screen this client for TB. There is no evidence to support the need to screen the client for
sickle cell disease, herpes zoster, or sick sinus syndrome.
C) The homeless client who abuses drugs is at risk for contracting tuberculosis. The nurse would
expect to screen this client for TB. There is no evidence to support the need to screen the client for
sickle cell disease, herpes zoster, or sick sinus syndrome.
D) The homeless client who abuses drugs is at risk for contracting tuberculosis. The nurse would
expect to screen this client for TB. There is no evidence to support the need to screen the client for
sickle cell disease, herpes zoster, or sick sinus syndrome.
Page Ref: 607
Cognitive Level: Applying
Client Need: Health Promotion and Maintenance
Nursing Process: Implementation
Learning Outcome: 3. Illustrate the nursing process in providing culturally competent care across the
life span for individuals with tuberculosis.
4) The nurse is planning care for a 90-year-old client who was recently diagnosed with tuberculosis.
The client lives alone in an apartment and will continue treatment at home. Which nursing diagnosis
is a priority for this client?
A) Ineffective Therapeutic Regimen Management
B) Deficient Knowledge
C) Ineffective Breathing Pattern
D) Risk for Injury
Answer: A
Explanation: A) The treatment regimen for tuberculosis requires that the client take many
medications, maintain nutrition, and be aware of potential side effects. Due to increased age and
normal forgetfulness, this client is at risk for ineffective treatment in the home. The client may have a
knowledge deficit but the priority is the treatment regimen. Since the client is being treated in the
home, there is not much risk for ineffective breathing. The client is at risk for injury because of age,
not TB.
B) The treatment regimen for tuberculosis requires that the client take many medications, maintain
nutrition, and be aware of potential side effects. Due to increased age and normal forgetfulness, this
client is at risk for ineffective treatment in the home. The client may have a knowledge deficit but the
priority is the treatment regimen. Since the client is being treated in the home, there is not much risk
for ineffective breathing. The client is at risk for injury because of age, not TB.
C) The treatment regimen for tuberculosis requires that the client take many medications, maintain
nutrition, and be aware of potential side effects. Due to increased age and normal forgetfulness, this
client is at risk for ineffective treatment in the home. The client may have a knowledge deficit but the
priority is the treatment regimen. Since the client is being treated in the home, there is not much risk
for ineffective breathing. The client is at risk for injury because of age, not TB.
D) The treatment regimen for tuberculosis requires that the client take many medications, maintain
nutrition, and be aware of potential side effects. Due to increased age and normal forgetfulness, this
client is at risk for ineffective treatment in the home. The client may have a knowledge deficit but the
priority is the treatment regimen. Since the client is being treated in the home, there is not much risk
for ineffective breathing. The client is at risk for injury because of age, not TB.
Page Ref: 614
Cognitive Level: Analyzing
Client Need: Physiological Integrity
Nursing Process: Planning
Learning Outcome: 4. Formulate priority nursing diagnoses appropriate for an individual with
tuberculosis.
5) An occupational health nurse is screening a new employee in a long-term care facility for
tuberculosis. The employee questions why purified protein derivative (PPD) testing is done twice.
Which is the most appropriate response by the nurse?
A) "Different medication is used in the second PPD."
B) "The treatment for TB is 6 months of medication, and we want to make sure the first results of the
first PPD were accurate."
C) "The first PPD was not interpreted in the correct time frame of 48-72 hours."
D) "There is an increased risk for a false-negative response for people who work in long-term care
facilities. The two-step is recommended to accurately screen for TB."
Answer: D
Explanation: A) PPD testing is done in a two-step process for people who work in long-term care
facilities because of the risk of false-negative responses. Treatment for TB for 6 months is not a reason
to complete the PPD twice. PPD testing is not done twice because different medication is used.
Evaluating the test at the wrong interval is not the reason that the PPD is done twice for long-term
care facility employees.
B) PPD testing is done in a two-step process for people who work in long-term care facilities because
of the risk of false-negative responses. Treatment for TB for 6 months is not a reason to complete the
PPD twice. PPD testing is not done twice because different medication is used. Evaluating the test at
the wrong interval is not the reason that the PPD is done twice for long-term care facility employees.
C) PPD testing is done in a two-step process for people who work in long-term care facilities because
of the risk of false-negative responses. Treatment for TB for 6 months is not a reason to complete the
PPD twice. PPD testing is not done twice because different medication is used. Evaluating the test at
the wrong interval is not the reason that the PPD is done twice for long-term care facility employees.
D) PPD testing is done in a two-step process for people who work in long-term care facilities because
of the risk of false-negative responses. Treatment for TB for 6 months is not a reason to complete the
PPD twice. PPD testing is not done twice because different medication is used. Evaluating the test at
the wrong interval is not the reason that the PPD is done twice for long-term care facility employees.
Page Ref: 611
Cognitive Level: Applying
Client Need: Health Promotion and Maintenance
Nursing Process: Implementation
Learning Outcome: 6. Plan evidence-based care for an individual with tuberculosis and the family in
collaboration with other members of the healthcare team.
6) The nurse is caring for a client who is receiving multiple drugs for treatment of tuberculosis. The
nurse teaches the client the rationale for the multiple-drug treatment and evaluates learning as
effective when the client makes which statement?
A) "Multiple drugs are necessary to develop immunity to tuberculosis."
B) "Multiple drugs are necessary because I became infected from an immigrant."
C) "Multiple drugs will be required as long as I am contagious."
D) "Multiple drugs are necessary because of the risk of resistance."
Answer: D
Explanation: A) Tuberculosis bacilli are likely to develop resistance to one drug, so multiple drugs
must be used. Treatment must be continued long after the client is no longer contagious. There is no
indication that the client contracted TB from an immigrant. Multiple drugs are used for all cases of
TB.
B) Tuberculosis bacilli are likely to develop resistance to one drug, so multiple drugs must be used.
Treatment must be continued long after the client is no longer contagious. There is no indication that
the client contracted TB from an immigrant. Multiple drugs are used for all cases of TB.
C) Tuberculosis bacilli are likely to develop resistance to one drug, so multiple drugs must be used.
Treatment must be continued long after the client is no longer contagious. There is no indication that
the client contracted TB from an immigrant. Multiple drugs are used for all cases of TB.
D) Tuberculosis bacilli are likely to develop resistance to one drug, so multiple drugs must be used.
Treatment must be continued long after the client is no longer contagious. There is no indication that
the client contracted TB from an immigrant. Multiple drugs are used for all cases of TB.
Page Ref: 611
Cognitive Level: Applying
Client Need: Physiological Integrity
Nursing Process: Implementation
Learning Outcome: 7. Evaluate expected outcomes for an individual with tuberculosis.
7) The nurse is caring for a client who has been admitted to the unit with tuberculosis. The client is
placed in isolation. To protect the caregivers and other clients on the unit, which type of isolation
room is most appropriate?
A) Single-door room with positive air flow (Air flows out of the room.)
B) Isolation room with an anteroom and negative air flow (Air flows into the room.)
C) Isolation room with an anteroom and normal airflow
D) Single-door room with normal airflow
Answer: B
Explanation: A) Patients with airborne infections such as meningococcemia, SARS, or TB are placed
in an isolation room with an anteroom and negative pressure airflow. Air flows into the room and is
vented in a special manner to prevent the organism from entering the rest of the unit. Positive flow
rooms are used for those clients who are immunosuppressed so that microorganisms from the unit
are not drawn into the room. Single-door isolation with normal airflow might be used for a client with
droplet or wound infection. Single-door rooms are not equipped to have positive or negative airflow.
B) Patients with airborne infections such as meningococcemia, SARS, or TB are placed in an isolation
room with an anteroom and negative pressure airflow. Air flows into the room and is vented in a
special manner to prevent the organism from entering the rest of the unit. Positive flow rooms are
used for those clients who are immunosuppressed so that microorganisms from the unit are not
drawn into the room. Single-door isolation with normal airflow might be used for a client with droplet
or wound infection. Single-door rooms are not equipped to have positive or negative airflow.
C) Patients with airborne infections such as meningococcemia, SARS, or TB are placed in an isolation
room with an anteroom and negative pressure airflow. Air flows into the room and is vented in a
special manner to prevent the organism from entering the rest of the unit. Positive flow rooms are
used for those clients who are immunosuppressed so that microorganisms from the unit are not
drawn into the room. Single-door isolation with normal airflow might be used for a client with droplet
or wound infection. Single-door rooms are not equipped to have positive or negative airflow.
D) Patients with airborne infections such as meningococcemia, SARS, or TB are placed in an isolation
room with an anteroom and negative pressure airflow. Air flows into the room and is vented in a
special manner to prevent the organism from entering the rest of the unit. Positive flow rooms are
used for those clients who are immunosuppressed so that microorganisms from the unit are not
drawn into the room. Single-door isolation with normal airflow might be used for a client with droplet
or wound infection. Single-door rooms are not equipped to have positive or negative airflow.
Page Ref: 616
Cognitive Level: Applying
Client Need: Safe and Effective Care Environment
Nursing Process: Implementation
Learning Outcome: 5. Summarize therapies used by interdisciplinary teams in the collaborative care
of an individual with tuberculosis.
8) The charge nurse for a medical-surgical nursing unit has been notified that a client with
tuberculosis is being transported to the unit. Which are the most appropriate actions for infection
prevention in this circumstance?
Select all that apply.
A) Stock the client's supply cart at the beginning of each shift.
B) Wear a mask and gown when caring for the client.
C) Have the client wear a mask when coming from admissions.
D) Perform hand hygiene only after leaving the room.
Answer: B, C
Explanation: A) Masks and gowns should be worn when caring for clients who do not reliably cover
their mouths when coughing. When a client has an airborne disease and must go elsewhere in the
hospital, the client must wear a mask. Supplies to prevent transmission of disease should be stocked
at the end of the shift so that adequate supplies will be available for the next healthcare provider.
Hand hygiene should be performed before and after client care.
B) Masks and gowns should be worn when caring for clients who do not reliably cover their mouths
when coughing. When a client has an airborne disease and must go elsewhere in the hospital, the
client must wear a mask. Supplies to prevent transmission of disease should be stocked at the end of
the shift so that adequate supplies will be available for the next healthcare provider. Hand hygiene
should be performed before and after client care.
C) Masks and gowns should be worn when caring for clients who do not reliably cover their mouths
when coughing. When a client has an airborne disease and must go elsewhere in the hospital, the
client must wear a mask. Supplies to prevent transmission of disease should be stocked at the end of
the shift so that adequate supplies will be available for the next healthcare provider. Hand hygiene
should be performed before and after client care.
D) Masks and gowns should be worn when caring for clients who do not reliably cover their mouths
when coughing. When a client has an airborne disease and must go elsewhere in the hospital, the
client must wear a mask. Supplies to prevent transmission of disease should be stocked at the end of
the shift so that adequate supplies will be available for the next healthcare provider. Hand hygiene
should be performed before and after client care.
Page Ref: 616
Cognitive Level: Applying
Client Need: Safe and Effective Care Environment
Nursing Process: Implementation
Learning Outcome: 2. Identify risk factors and prevention methods associated with tuberculosis.
9) A nurse is caring for a client with tuberculosis who is taking Rifampin for treatment of the disease.
Of which nursing considerations should the nurse be aware regarding this medication?
Select all that apply.
A) Administer with meals to reduce gastrointestinal side effects.
B) Record a baseline visual examination before therapy.
C) Administer on an empty stomach.
D) Administer by deep intramuscular injection into a large muscle mass.
E) Monitor CBC, liver function studies, and renal function studies for evidence of toxicity.
Answer: C, E
Explanation: A) Rifampin is an oral antituberculosis medication that should be administered on an
empty stomach. The nurse should monitor the CBC, liver function studies, and renal function studies.
A baseline visual examination before therapy is necessary with ethambutol, another antituberculosis
medication.
B) Rifampin is an oral antituberculosis medication that should be administered on an empty stomach.
The nurse should monitor the CBC, liver function studies, and renal function studies. A baseline
visual examination before therapy is necessary with ethambutol, another antituberculosis medication.
C) Rifampin is an oral antituberculosis medication that should be administered on an empty stomach.
The nurse should monitor the CBC, liver function studies, and renal function studies. A baseline
visual examination before therapy is necessary with ethambutol, another antituberculosis medication.
D) Rifampin is an oral antituberculosis medication that should be administered on an empty stomach.
The nurse should monitor the CBC, liver function studies, and renal function studies. A baseline
visual examination before therapy is necessary with ethambutol, another antituberculosis medication.
E) Rifampin is an oral antituberculosis medication that should be administered on an empty stomach.
The nurse should monitor the CBC, liver function studies, and renal function studies. A baseline
visual examination before therapy is necessary with ethambutol, another antituberculosis medication.
Page Ref: 613
Cognitive Level: Applying
Client Need: Physiological Integrity
Nursing Process: Implementation
Learning Outcome: 5. Summarize therapies used by interdisciplinary teams in the collaborative care
of an individual with tuberculosis.
10) The nurse caring for a homeless client at risk for tuberculosis will include which symptoms of the
disease when educating the client?
Select all that apply.
A) Fatigue
B) Low-grade morning fever
C) Productive cough that later turns to a dry, hacking cough
D) Weight loss
E) Night sweats
Answer: A, D, E
Explanation: A) Manifestations of tuberculosis often develop insidiously and are initially nonspecific.
Fatigue, weight loss, diminished appetite, low-grade afternoon fever, and night sweats are common. A
dry cough develops, which later becomes productive of purulent and/or blood-tinged sputum. It is
often at this stage that the client first seeks medical attention.
B) Manifestations of tuberculosis often develop insidiously and are initially nonspecific. Fatigue,
weight loss, diminished appetite, low-grade afternoon fever, and night sweats are common. A dry
cough develops, which later becomes productive of purulent and/or blood-tinged sputum. It is often
at this stage that the client first seeks medical attention.
C) Manifestations of tuberculosis often develop insidiously and are initially nonspecific. Fatigue,
weight loss, diminished appetite, low-grade afternoon fever, and night sweats are common. A dry
cough develops, which later becomes productive of purulent and/or blood-tinged sputum. It is often
at this stage that the client first seeks medical attention.
D) Manifestations of tuberculosis often develop insidiously and are initially nonspecific. Fatigue,
weight loss, diminished appetite, low-grade afternoon fever, and night sweats are common. A dry
cough develops, which later becomes productive of purulent and/or blood-tinged sputum. It is often
at this stage that the client first seeks medical attention.
E) Manifestations of tuberculosis often develop insidiously and are initially nonspecific. Fatigue,
weight loss, diminished appetite, low-grade afternoon fever, and night sweats are common. A dry
cough develops, which later becomes productive of purulent and/or blood-tinged sputum. It is often
at this stage that the client first seeks medical attention.
Page Ref: 609
Cognitive Level: Understanding
Client Need: Physiological Integrity
Nursing Process: Implementation
Learning Outcome: 1. Describe the pathophysiology, etiology, clinical manifestations, and direct and
indirect causes of tuberculosis.
Exemplar 9.8 Urinary Tract Infection

1) A 26-year-old female client is admitted to the hospital with a diagnosis of kidney stones. The
physician orders IV fluids, x-rays, blood work, and a Foley catheter for the client. The nurse is caring
for the client 3 days after admission and takes morning vital signs of 101°F, heart rate 92, respirations
25, and blood pressure 120/80. The urinary output has decreased, and the urine is cloudy and dark
amber. What should the nurse suspect is occurring with the client?
A) The client has passed the kidney stones.
B) The client is in acute renal failure.
C) The client has developed a respiratory infection.
D) The client has a probable nosocomial urinary tract infection.
Answer: D
Explanation: A) The client has developed a urinary tract infection that was not present upon
admission. This is a healthcare-associated infection (HAI) and is likely due to poor technique when
placing the Foley catheter. There is no evidence of a respiratory infections, renal failure, or the
passage of kidney stones.
B) The client has developed a urinary tract infection that was not present upon admission. This is a
healthcare-associated infection (HAI) and is likely due to poor technique when placing the Foley
catheter. There is no evidence of a respiratory infections, renal failure, or the passage of kidney
stones.
C) The client has developed a urinary tract infection that was not present upon admission. This is a
healthcare-associated infection (HAI) and is likely due to poor technique when placing the Foley
catheter. There is no evidence of a respiratory infections, renal failure, or the passage of kidney
stones.
D) The client has developed a urinary tract infection that was not present upon admission. This is a
healthcare-associated infection (HAI) and is likely due to poor technique when placing the Foley
catheter. There is no evidence of a respiratory infections, renal failure, or the passage of kidney
stones.
Page Ref: 621
Cognitive Level: Analyzing
Client Need: Physiological Integrity
Nursing Process: Assessment
Learning Outcome: 1. Describe the pathophysiology, etiology, and clinical manifestations of urinary
tract infection.
2) The nurse for a urology clinic is planning an in-service about urinary infections for a group of
novice nurses. What should be included in the presentation?
A) Men are most likely to experience descending urinary tract infections.
B) Straight catheterization is the only way to evaluate for the presence of a urinary tract infection.
C) Women require a shorter course of antibiotic therapy to manage a urinary tract infection than men.
D) The rate of urinary tract infections is similar between men and women.
Answer: C
Explanation: A) Men will usually be prescribed a longer course of therapy to manage a urinary tract
infection. They have a longer urethra. When they are infected with an infection to the system, it is
typically more complicated than in a woman. Descending infections are less common than ascending
infections of the urinary tract. A clean-catch midstream specimen is a reliable means to obtain a urine
specimen to check for the presence of infection. Women have a higher rate of urinary tract infection
than men.
B) Men will usually be prescribed a longer course of therapy to manage a urinary tract infection. They
have a longer urethra. When they are infected with an infection to the system, it is typically more
complicated than in a woman. Descending infections are less common than ascending infections of
the urinary tract. A clean-catch midstream specimen is a reliable means to obtain a urine specimen to
check for the presence of infection. Women have a higher rate of urinary tract infection than men.
C) Men will usually be prescribed a longer course of therapy to manage a urinary tract infection. They
have a longer urethra. When they are infected with an infection to the system, it is typically more
complicated than in a woman. Descending infections are less common than ascending infections of
the urinary tract. A clean-catch midstream specimen is a reliable means to obtain a urine specimen to
check for the presence of infection. Women have a higher rate of urinary tract infection than men.
D) Men will usually be prescribed a longer course of therapy to manage a urinary tract infection. They
have a longer urethra. When they are infected with an infection to the system, it is typically more
complicated than in a woman. Descending infections are less common than ascending infections of
the urinary tract. A clean-catch midstream specimen is a reliable means to obtain a urine specimen to
check for the presence of infection. Women have a higher rate of urinary tract infection than men.
Page Ref: 624
Cognitive Level: Applying
Client Need: Health Promotion and Maintenance
Nursing Process: Implementation
Learning Outcome: 2. Identify risk factors and prevention methods associated with urinary tract
infection.
3) The nurse is giving discharge instructions to the family of an elderly client with a history of urinary
tract infection (UTI). The family asks what the early symptoms of a urinary tract infection are so that
they can monitor the client. Which early symptom of a UTI should the nurse teach the family?
A) Urinary urgency
B) Blood in the urine
C) Urinary frequency
D) Alteration in cognition
Answer: D
Explanation: A) The early manifestations of a urinary infection differ in a senior from those seen in
younger adults. Older adult clients will often be asymptomatic until there are changes in the level of
consciousness.
B) The early manifestations of a urinary infection differ in a senior from those seen in younger adults.
Older adult clients will often be asymptomatic until there are changes in the level of consciousness.
C) The early manifestations of a urinary infection differ in a senior from those seen in younger adults.
Older adult clients will often be asymptomatic until there are changes in the level of consciousness.
D) The early manifestations of a urinary infection differ in a senior from those seen in younger adults.
Older adult clients will often be asymptomatic until there are changes in the level of consciousness.
Page Ref: 621
Cognitive Level: Applying
Client Need: Health Promotion and Maintenance
Nursing Process: Implementation
Learning Outcome: 3. Illustrate the nursing process in providing culturally competent care across the
life span for individuals with urinary tract infection.
4) A preadolescent student asks the school nurse to explain why the student keeps getting urinary
tract infections. The school nurse determines that the diagnosis of Deficient Knowledge is appropriate
based upon the student's response to which question?
A) "When was your last UTI?"
B) "How often do you shower?"
C) "Do you have a family history of urinary problems?"
D) "In what direction do you wipe after a bowel movement?"
Answer: D
Explanation: A) The most important teaching to provide females is always to wipe the peri-
anal/genital area from front to back. E-coli are the most common micro-organisms responsible for
urinary tract infections and can easily be dragged into the urethral orifice by wiping from the anus to
the urethra after defecation. Females do have a shorter urethra compared with males and are more
susceptible to urinary tract infections for this reason. Personal hygiene practices, such as cleansing
after a bowel movement, are a priority teaching instruction to include for this client. Urinary tract
infections are not associated with families or genetics.
B) The most important teaching to provide females is always to wipe the peri-anal/genital area from
front to back. E-coli are the most common micro-organisms responsible for urinary tract infections
and can easily be dragged into the urethral orifice by wiping from the anus to the urethra after
defecation. Females do have a shorter urethra compared with males and are more susceptible to
urinary tract infections for this reason. Personal hygiene practices, such as cleansing after a bowel
movement, are a priority teaching instruction to include for this client. Urinary tract infections are not
associated with families or genetics.
C) The most important teaching to provide females is always to wipe the peri-anal/genital area from
front to back. E-coli are the most common micro-organisms responsible for urinary tract infections
and can easily be dragged into the urethral orifice by wiping from the anus to the urethra after
defecation. Females do have a shorter urethra compared with males and are more susceptible to
urinary tract infections for this reason. Personal hygiene practices, such as cleansing after a bowel
movement, are a priority teaching instruction to include for this client. Urinary tract infections are not
associated with families or genetics.
D) The most important teaching to provide females is always to wipe the peri-anal/genital area from
front to back. E-coli are the most common micro-organisms responsible for urinary tract infections
and can easily be dragged into the urethral orifice by wiping from the anus to the urethra after
defecation. Females do have a shorter urethra compared with males and are more susceptible to
urinary tract infections for this reason. Personal hygiene practices, such as cleansing after a bowel
movement, are a priority teaching instruction to include for this client. Urinary tract infections are not
associated with families or genetics.
Page Ref: 625
Cognitive Level: Analyzing
Client Need: Health Promotion and Maintenance
Nursing Process: Planning
Learning Outcome: 4. Formulate priority nursing diagnoses appropriate for an individual with
urinary tract infection.
5) The nurse has been invited to present a program at the local PTA meeting on recognizing and
preventing illness in children. Which symptoms of urinary tract infections in preschool age children
should the nurse include?
Select all that apply.
A) Urinary urgency
B) Elevated blood pressure
C) Dysuria
D) Fever
E) Headache
Answer: A, C, D
Explanation: A) Clinical manifestations of a urinary tract infection (UTI) in a preschool-age child
include fever, urgency, and dysuria. Headache and elevated blood pressure are not clinical
manifestations of a UTI for a preschool-age child.
B) Clinical manifestations of a urinary tract infection (UTI) in a preschool-age child include fever,
urgency, and dysuria. Headache and elevated blood pressure are not clinical manifestations of a UTI
for a preschool-age child.
C) Clinical manifestations of a urinary tract infection (UTI) in a preschool-age child include fever,
urgency, and dysuria. Headache and elevated blood pressure are not clinical manifestations of a UTI
for a preschool-age child.
D) Clinical manifestations of a urinary tract infection (UTI) in a preschool-age child include fever,
urgency, and dysuria. Headache and elevated blood pressure are not clinical manifestations of a UTI
for a preschool-age child.
E) Clinical manifestations of a urinary tract infection (UTI) in a preschool-age child include fever,
urgency, and dysuria. Headache and elevated blood pressure are not clinical manifestations of a UTI
for a preschool-age child.
Page Ref: 622-623
Cognitive Level: Applying
Client Need: Health Promotion and Maintenance
Nursing Process: Implementation
Learning Outcome: 6. Plan evidence-based care for an individual with urinary tract infection and the
family in collaboration with other members of the healthcare team.
6) The nurse is providing discharge teaching for a female client with a urinary tract infection. The
client has been prescribed a 3-day course of oral trimethoprim-sulfamethoxazole (TMP-SMZ). What
client statement indicates that teaching has been effective?
A) "I will return within 10 days for a follow-up urine culture."
B) "I will practices Kegel exercises on daily basis."
C) "I will increase my intake of fluids, especially citrus juice."
D) "I will only wear 100% cotton underwear."
Answer: A
Explanation: A) It is essential to validate eradication of infection with a follow-up culture that is
negative. Doing Kegel exercises and wearing cotton underwear are both useful in prevention of future
urinary tract infections, but they are not the best evaluation of effectiveness of teaching. Citrus juices
will not increase acidity of urine and therefore are not recommended when a client has a UTI.
B) It is essential to validate eradication of infection with a follow-up culture that is negative. Doing
Kegel exercises and wearing cotton underwear are both useful in prevention of future urinary tract
infections, but they are not the best evaluation of effectiveness of teaching. Citrus juices will not
increase acidity of urine and therefore are not recommended when a client has a UTI.
C) It is essential to validate eradication of infection with a follow-up culture that is negative. Doing
Kegel exercises and wearing cotton underwear are both useful in prevention of future urinary tract
infections, but they are not the best evaluation of effectiveness of teaching. Citrus juices will not
increase acidity of urine and therefore are not recommended when a client has a UTI.
D) It is essential to validate eradication of infection with a follow-up culture that is negative. Doing
Kegel exercises and wearing cotton underwear are both useful in prevention of future urinary tract
infections, but they are not the best evaluation of effectiveness of teaching. Citrus juices will not
increase acidity of urine and therefore are not recommended when a client has a UTI.
Page Ref: 624
Cognitive Level: Applying
Client Need: Health Promotion and Maintenance
Nursing Process: Evaluation
Learning Outcome: 7. Evaluate expected outcomes for an individual with urinary tract infection.
7) The physician has ordered an indwelling urinary catheter for a client with urinary retention. Which
intervention, along with strict aseptic technique, will decrease the risk of infection for this procedure?
A) Irrigating the catheter with sterile saline on a daily basis
B) Instructing the client to void around the catheter
C) Using an anesthetic lubricating gel during insertion
D) Inflating the balloon while the catheter is in the urethra
Answer: C
Explanation: A) Unless contraindicated, the additional step of using an anesthetic lubricating gel
promotes comfort and protects fragile urethral tissues from trauma, and therefore reduces risk for a
catheter-associated UTI. Irrigating the catheter should not be done because it can introduce infection
by allowing bacteria to enter the closed urinary drainage system. Although voiding around the
catheter will decrease bladder spasms, it will not help reduce infection. The balloon is not inflated
until the catheter is in the bladder, in order to prevent trauma to the urethra and therefore decrease
the risk of infection.
B) Unless contraindicated, the additional step of using an anesthetic lubricating gel promotes comfort
and protects fragile urethral tissues from trauma, and therefore reduces risk for a catheter-associated
UTI. Irrigating the catheter should not be done because it can introduce infection by allowing bacteria
to enter the closed urinary drainage system. Although voiding around the catheter will decrease
bladder spasms, it will not help reduce infection. The balloon is not inflated until the catheter is in the
bladder, in order to prevent trauma to the urethra and therefore decrease the risk of infection.
C) Unless contraindicated, the additional step of using an anesthetic lubricating gel promotes comfort
and protects fragile urethral tissues from trauma, and therefore reduces risk for a catheter-associated
UTI. Irrigating the catheter should not be done because it can introduce infection by allowing bacteria
to enter the closed urinary drainage system. Although voiding around the catheter will decrease
bladder spasms, it will not help reduce infection. The balloon is not inflated until the catheter is in the
bladder, in order to prevent trauma to the urethra and therefore decrease the risk of infection.
D) Unless contraindicated, the additional step of using an anesthetic lubricating gel promotes comfort
and protects fragile urethral tissues from trauma, and therefore reduces risk for a catheter-associated
UTI. Irrigating the catheter should not be done because it can introduce infection by allowing bacteria
to enter the closed urinary drainage system. Although voiding around the catheter will decrease
bladder spasms, it will not help reduce infection. The balloon is not inflated until the catheter is in the
bladder, in order to prevent trauma to the urethra and therefore decrease the risk of infection.
Page Ref: 627
Cognitive Level: Applying
Client Need: Safe and Effective Care Environment
Nursing Process: Implementation
Learning Outcome: 5. Summarize therapies used by interdisciplinary teams in the collaborative care
of an individual with urinary tract infection.
8) The nurse in an urgent care center assesses a 40-year-old female client who has presented with a
fever of 101.2°F and complaints of painful urination. What should the nurse ask to elicit further data
that indicates cystitis?
A) "Do you have any symptoms of menopause?"
B) "How long have you had a fever, and have you had chills with this?"
C) "Do you have any upper abdominal pain or cramping?"
D) "What color is your urine?"
Answer: D
Explanation: A) The classic symptoms of cystitis include dysuria or painful urination, urinary
frequency and urgency, and bloody urine or hematuria. A 40-year-old client who presents with a fever
and painful urination is not experiencing symptoms of menopause. Asking about fever and chills is
not specific to suspected cystitis and does not elicit information related to urinary symptoms. Pain
from cystitis is typically suprapubic, not upper abdominal.
B) The classic symptoms of cystitis include dysuria or painful urination, urinary frequency and
urgency, and bloody urine or hematuria. A 40-year-old client who presents with a fever and painful
urination is not experiencing symptoms of menopause. Asking about fever and chills is not specific to
suspected cystitis and does not elicit information related to urinary symptoms. Pain from cystitis is
typically suprapubic, not upper abdominal.
C) The classic symptoms of cystitis include dysuria or painful urination, urinary frequency and
urgency, and bloody urine or hematuria. A 40-year-old client who presents with a fever and painful
urination is not experiencing symptoms of menopause. Asking about fever and chills is not specific to
suspected cystitis and does not elicit information related to urinary symptoms. Pain from cystitis is
typically suprapubic, not upper abdominal.
D) The classic symptoms of cystitis include dysuria or painful urination, urinary frequency and
urgency, and bloody urine or hematuria. A 40-year-old client who presents with a fever and painful
urination is not experiencing symptoms of menopause. Asking about fever and chills is not specific to
suspected cystitis and does not elicit information related to urinary symptoms. Pain from cystitis is
typically suprapubic, not upper abdominal.
Page Ref: 621
Cognitive Level: Applying
Client Need: Physiological Integrity
Nursing Process: Assessment
Learning Outcome: 1. Describe the pathophysiology, etiology, clinical manifestations, and direct and
indirect causes of urinary tract infection.

You might also like